Download as pdf or txt
Download as pdf or txt
You are on page 1of 100

Module1:

The Nature of Mathematics


Introduction
M athematics has been part of human’s everyday life. It is everywhere. It is on the things
we create and admire, in the activities we do, in the decisions we make. Mathematics is
present in everything that surrounds us. It could be seen on the patterns and sequences
exhibited by nature from its landscapes, plants and animals, including humans to series of
events and phenomena.
Recognizing these patterns gives us understanding on how nature works and helps
create things that human may benefit from. Through this, advancement of science and
technology could be done. Mathematics could help predict the behavior of nature especially
the naturally-occurring phenomena and create intervention to safeguard the lives of human.
This could provide necessary answers to the questions that puzzles humankind that may
create a much better world.
In this chapter, we shall look at mathematics as a science of pattern and order to
encourage you to go beyond the typical understanding of mathematics as merely as science
of calculations but as a source of aesthetics in patterns of nature and a rich language in itself
governed by logic and reasoning.

Learning Outcomes:
At the end of this chapter, you are expected to:

 Identify the different types of patterns in mathematics.


 Use inductive and deductive reasoning to predict patterns.
 Write expressions and statements in mathematical symbols.
 Explore on the different patterns found in objects, plants, and animals that
can be modelled mathematically.

1
LESSON 1:
NATURE AND CHARACTERISTICS OF
MATHEMATICS

What is Mathematics?

Traditionally, mathematics is presented deductively at school. And is often perceived


as well structured and problems are algorithmically approached. Mathematics is derived from
the ancient word manthnein meaning “to learn”. The Greek root mathesis means “knowledge”
or its form mathema meaning science, knowledge, or learning, and mathematikos or
mathemata means “fond of learning”. These might have been the notion of the early
mathematicians and philosophers that is why they continue to seek for knowledge and the
truth.

Hence, what is really mathematics? What is it exactly that mathematicians do when


they are doing mathematics?

Encyclopedia Britannica defines mathematics as the “science of structure, order, and


relations that has evolved from the elemental practices of counting, measuring, and describing
the shapes and characteristics of objects.” This definition is one of the closest to mathematics
that is evident in the modern world today.

Characteristics of Mathematics

What are the characteristics of mathematics? Among the characteristics of


mathematics are:

Classification

Logical
Generalization
Sequence

Characteristcs
Applicability Structure
of Mathematics

Mathematical
Precision and
language and
Accuracy
symbilism

Abstractness 2
Classification

Within logical-mathematical knowledge is the process of classification which represents


the first step towards learning more complex mathematical concepts. Classification brings
about a series of mental relations through which objects are grouped according to similarities
and differences depending on specific criteria such as size, color, shape, etc.

Logical Sequence

Ideas in mathematics need to flow in order that makes sense. Logic patterns are related
to geometric and number patterns. It helps us classify objects or figures. One kind of logic
sequence deal with characteristics of various objects. Another kind deals with orders: there
is a sequence of objects and a pattern in the attributes the objects possess.

Structure

A structure on a set is an additional mathematical object that is related to that given set in
some particular characteristic or manner, and attaches (or relates) to that set to endow it with
some additional meaning or significance. The structure on a particular mathematical set will
allow mathematicians to study the set further and find its relationship with objects.

Precision and Accuracy

Accuracy is how close a measured value is equal to the actual value. It is the degree to
which a given quantity is correct and free from error. Precision, on the other hand, is how close
the measured values are to each other.

Abstractness

Abstractness can be anything from strings of numbers or letters to geometric figures to


measurements to set of equations. Inquiries such as “Does the interval between composite
numbers from a pattern?” “Is there an expression for the change in the surface area of any
regular solid as its volume approaches zero?”; needs investigatory approaches of patterns.

Symbolism

The language of mathematics is the system used by the mathematicians to communicate


mathematical ideas using symbols instead of words. This language is uniquely constructed in
such a way that all mathematicians understand symbiotic notations and mathematical
formulas.

Applicability

The applicability of mathematics can lie anywhere on a spectrum from the completely
simple to utterly complex. Mathematics can be used in all fields of human endeavour.
According to Kant, “in any special doctrine of nature, there can be only as much proper
science as there is mathematics therein.” Many agree that the problem of understanding the
significant link between mathematics and modern science is an interesting and significant
challenge for the philosopher of mathematics.

3
Generalizations

Making generalizations is fundamental to mathematics. It is a skill that must be


developed among students. It is of vital importance in a functioning society. The Webster
Dictionary defines generalization as the process of “finding and singling out [of properties]
in a whole class of similar objects.”

LESSON 2
MATHEMATICS AS A SCIENCE OF PATTERNS

A pattern is a visible regularity in the universe or in a man-made design. As such, the


elements of a pattern repeat in a predictable manner. Patterns, in a mathematical sense, refer
to the study of “tilings” “and wall-paper symmetries.” This connotes order, regularity, and
lawfulness. In the modern world, the patterns of counting, measuring, reasoning, motion,
shape, position, and prediction, reveal the powerful influence mathematics has over people’s
perception of reality.

Pattern recognition is a key determinant of logical, verbal, mathematical, and spatial


abilities. The most commonly used patterns today: the logic patterns, symbol patterns, number
patterns, and word patterns.

Logic Patterns

Logic patterns deal with the characteristics of various objects, order, sequence and
some possess similar attributes. These types of pattern are common in aptitude tests. To
construct or solve a pattern: 1) find the rule for the pattern, 2) understand the nature of the
sequence, and 3) analyze the difference between two successive terms.

Example 1:

Answer: The lines rotate in a counter-


clockwise manner from the uppermost box
down to the fourth box. The fourth box in the
preceding object is the first box in the next
object. The pattern continues so the answer
is B.

4
Example 2:

Answer: The second figure is just the double of


the first figure, so it has two circles. The third figure
is the double of the second figure, thus it has four
circles. The fourth must be the double of the third
so it should have eight. Answer is D.

Example 3:

Answer: Look at the movements


of the arrows inside the triangles.
The arrow starts from a vertex and
the arrowhead points towards the
opposite side. So, the missing
figure should be B.

Example 4:

If the word MODERN can be encrypted as OQFGTP, how can you code the
word WORLD?

a. YQSNF c. YQUNF

b. YQTNF d. YQPNF

Answer: The correct answer is b. Each letter in the original word MODERN is
forwarded two places (+2) in the alphabetical order to get the encrypted word.

Example 5:

In some code language, the word PATTERNS can be written TRTRTRTR, how
would you write ALGEBRA in the same code language?

a. RGGRRRG c. GRGRGRG

b. GRRGGR d. RRGGRR

Answer: The correct answer is c. Each letter in the third, sixth, ninth, etc. (multiples of
3) positions is reported according to the length of the given word.

5
Number Patterns

A number oattern is a list or set of numbers that follow a certain arrangement. It is


where the numbers are arranged in either increasing or decreasing order, and have with it
basic operations of mathematics or a certain series of arithmetical operation like addition or
subtraction repeatedly done. The first step in determining the rule that defines the pattern is to
look for differences between the two consecutive numbers. The number pattern helps make a
generalization of how the numbers are arranged in a sequence. If there is no logic (addition,
subtraction, multiplication, division, squares, cubes, primes, etc) in the differences, find other
operations used in the pattern.

Try these examples; guess what the next number in the sequence.

Example 6:

11, 13, 17, 19, 23, ?

Answer: Notice that the differences between two consecutive numbers are 2, 4, 2, 4.
However, this is not a pattern. Look upon the numbers again and guess what? They
are all consecutive prime numbers. So the next number must be 29.

Example 7:

5, 7, 10, 15, 22, ?

Answer: The differences between each pair of consecutive terms in the sequence are
as follows: 2, 3, 5, 7. These primes are added to the previous number to get the next
number. 5+2=7, 7+3=10, 10+5=15, 15+7=22. So you know that the next prime is 11.
Hence, the next number in the pattern is 22+11=33.

Example 8:

5, 6, 10, 19, 35, ?

Answer: The differences between each number is 1, 4, 9, 16. These numbers are
squares of 1, 2, 3, and 4. So, the difference between 35 and the next number should
be the square of 5, which is 25. Thus, 35+25=60.

Word Patterns

Patterns can also be found in language. The metrical patterns of poems and the
syntactic patterns of how nouns are made plural or how verbs are changed to past tense are
both word patterns. Word patterns are used in decoding like: consonant blends (words with
a group of two or three consonants that each make its own sound: grow, blend, sleeve, stair,
sweet, free, blood, clothes), consonant digraphs (words with two or three letters that come
together to create a single sound, e.g. chest, shop, sheep, brush, shirt, shade), vowel
diphthongs (vowels that glide in the middle: boil, now, soil, brown, cloud, rain, meat, bout,
cow), and vowel digraphs (a spelling pattern in which two or more adjoining letters represent
a single vowel sound: school, clean, each, feet, moon, cheese)

6
LESSON 3
THE FIBONACCI SEQUENCE

The Finonacci Sequence

Leonardo of Pisa is also known as


Fibonacci (1170 – 1250). He is one of the best –
known mathematicians of medieval Europe. In
1202, after a trip that took him to several Aran and
Eastern countries, Fibonacci wrote the book Liber
Abaci. In this book Fibonacci explained why Hindu
– Arabic numeration system that he had learned
about the his travels was a more sophisticated and
Source: artifacts.com
efficient system than the Roman numeration
system.This book also contains a problem created by Fibonacci that concerns the birth of
rabbits. Here is the statement of Fibonacci’s rabbit problem.

At the beginning of a month, you are given a pair of newborn rabbits. After a month the
rabbits have produced no offspring; however, every month thereafter the pair of rabbits
produces another pair of rabbits. The offspring reproduce in exactly the same manner. If none
of the rabbits dies, how many pairs of rabbits will be there be at the start of each succeeding
month?

The solution of this problem is a sequence of numbers that we now calleniscuola.net


Source: the Fibonacci
sequence.

The Fibonacci sequence is the series of numbers:

1, 1, 2, 3, 5, 8, 13, 21, 34….

7
The next number is found by adding up the two numbers before it:

The 2 is found by adding the two numbers before it (1+1),

The 3 is found by adding the two numbers before it (2+1),

And so on!

Remember!
The Fibonacci sequence can be written as a rule as:

𝑋𝑛 = 𝑋𝑛 − 1 + 𝑋𝑛 − 2
where:
𝑋𝑛 is term number “n” in the Fibonacci sequence

Example 9: Find the seventh and eighth Fibonacci numbers.

Solution:

For seventh Fibonacci number:

𝑋𝑛 = 𝑋𝑛 − 1 + 𝑋𝑛 − 2

𝑋7 = 𝑋7 − 1 + 𝑋7 − 2

𝑋7 = 𝑋6 + 𝑋5

𝑋7 = 8 + 5

𝑋7 = 13

For eight Fibonacci number:

𝑋𝑛 = 𝑋𝑛 − 1 + 𝑋𝑛 − 2

𝑋8 = 𝑋8 − 1 + 𝑋8 − 2

𝑋8 = 𝑋7 + 𝑋6

𝑋8 = 13 + 8

𝑋8 = 21

When we take any two successive Fibonacci numbers, their ratio is very close to the
Golden Ratio “φ” which is approximately 1.618034… In fact, the bigger the pair of Fibonacci
numbers, the closer the approximation. For example,

A B B/A

8
2 3 1.5

3 5 1.66666…

5 8 1.6

… … …

144 233 1.618055556…

233 377 1.618025751

It is even more surprising that we can calculate any Fibonacci number using the Golden
Ratio:

Remember!
Fibonacci number using the Golden Ratio:

𝝋𝒏 − (𝟏 − 𝝋)𝒏
𝒙𝒏 = 𝒐𝒓 𝒙𝒏 = 𝒙𝒏−𝟏 × 𝝋
√𝟓

Example 10. Find the sixth Fibonacci number.

Solution:

𝝋𝒏 − (𝟏 − 𝝋)𝒏
𝒙𝒏 =
√𝟓

1.618034 …6 − (1 − 1.618034 … )6
𝑥6 =
√5

𝑥6 = 8.00000033 ≈ 8

or

𝒙𝒏 = 𝒙𝒏−𝟏 × 𝝋

𝑥6 = 𝑥6−1 × 1.618034 …

𝑥6 = 5 × 1.618034 …

𝑥6 = 8.09017 ≈ 8

9
LESSON 4
MATHEMATICS FOR OUR WORLD

Millions of patterns can be found in the environment. These patterns occur in various
forms and in different contexts which can be modelled mathematically. Some examples are
symmetries, trees, spirals, waves, tessellations, stripes, meanders, cracks, and many more.

The German biologist Ernst Haeckel painted marine organisms to show their
symmetry. Scottish biologist D’Archy Thompson studied the growth patterns in plants and
animals. Beehives are one of the examples of patterns in nature that can be modelled
mathematically. They are made of walls of the same size, enclosing small hexagonal cells
where honey and pollen are stored and bees are raised. Honeycombs are examples of
wallpaper symmetry.

Scientists theorize that sunflowers can pack in the maximum number of seeds if each
seed is separated by an irrational-numbered angle. In mathematics, the most irrational-number
is known as the Golden Ratio, or Phi. It is a mathematical ratio that’s commonly found in
nature. It can be used to create visually-pleasing, organic-looking compositions in your design
projects or artwork.

Here are some examples of patterns in nature.

Source: curiodyssey.org Source: blog.csiro.au

Honeycomb Animal Skin

Source: viewbug.com Source: kidsdiscover.com

Plant Trees

10
Source: panachallenge.com Source: thescienceexplorer.com

Fruit Flower

Source: pinterest.com
Sand
Source: iphone6papers.com

Water

It could be noted that mathematics exists in the natural world. It is specifically on how
nature behaves and the patterns it follows through logical and mathematical structures. It is
important that these concepts and ideas be used to further advance what we know and
provides substantial and effective solutions to some underlying problems we faced nowadays.

Mathematics is an important tool in generating models, analysis and interpretations


that could be used in predicting the possible outcomes of a certain event. Like for examples,
to predict the probability of a storm to occur, the possible return period of an event having the
same magnitude, determine the trend of social media users, predict the increase of the world’s
population after 10 years, and others. This may give a clearer picture to innovators, scientists,
engineers and other professionals, even common people, on the possible scenarios that may
happen. This may lead to creating strategies to lessen the impact and several interventions to
addressed the possible outcomes.

In recent years, global warming and climate change, are hot topics that leads to
several debates, protests and legislations. Scientists warns that if man will not change his
behavior towards the environment, the effects of climate change is inevitable. Through the use
of mathematics, man is able to exert control over himself and possibly reverses the effects of
climate change on nature.

We have identified some of the applications of mathematics and that it plays a bigger
role in the societal context. We know that mathematics is with us in everything we do. Although

11
we may not notice it. Knowledge of mathematics is helpful in every endeavor. So, it is better
to embrace it to have a better understanding of the world we live in and be part of the crusade
in protecting and conserving our one and only natural world.

Source: Grading Rubrics for Reflection Journal Entries retrieved from


https://people.richland.edu/fbrenner/syllabus/reflectrubric.html

Module 2:
Speaking Mathematically

Introduction
M athematics can be thought of as a language filled with vocabulary, symbols, and
sentence structures. Learning both mathematics and the language of mathematics is a
challenge for all. These can make things difficult for all who wish to relate math to their
everyday language and experiences. And like any language, we have to speak math
proficiently in order to use it efficiently. With these, this chapter is designed to introduce you
to the language of sets and relations and functions. As you work through this module, you will
be introduced to different notations and concepts on sets, relations, and functions that will help
building and deepen your knowledge in learning mathematics.

Learning Outcomes:
At the end of this chapter, you are expected to:

 Describe sets.
 Solve problems involving operation of sets.
 Differentiate function and relation.
 Recognize relation that are not function.
 Determine the domain and range of a relation.

12
LESSON 1:
THE LANGUAGE OF SETS

Set

The concept of set is basic and it is impossible for us to define what set is. Set is simply
described in terms of the properties of those objects that belong to the set. Hence, set is
described as any group or collection of objects whose properties are well defined. This means
two things: the objects in the set, called elements must be distinguishable from each other
and description of the elements must be clear and unambiguous.
Set can be identified or named with the use of any capital letter or symbols. Elements
of sets are written with small letters or even the name of object itself, separated by commas,
within its braces. The symbol ∈ is utilized to indicate the membership of an obejct in a set
(element) and the symbol for not being a member or does not belong in a set (not an
element).

Example 1: Determine whether each statement is set or not. If set, list down the
elements.

1. The set of all positive numbers less than 8.


2. The set of months with 30 days.
3. The set of even numbers divisible by 3 and less than 50.
4. The set of tall people.
5. The set of nice cars.

Solution:

1. The statement is a set.


N = {1, 2, 3, 4, 5, 6, 7}

2. The statement is a set.


M = { September, November, April, June}

3. The statement is a set.


S = {6, 12, 18, 24, 30, 36, 42, 48}

4. The statement is not a set because people tend to disagree what “tall” means. The
word “tall” is not precise.
5. Same with number 4, the statement is not a set because people would not have
the same perception of what “nice” car is. The word “nice” is not precise.

Example 2: Determine whether each statement is true or false.

1. 6 is an element of all positive numbers less than 8.

13
2. 8 ∈ { 4, 5, 6, 7}
3. – 6 ∈ N
4. ½ Z

Solution:

1. Since 6 is an element of positive numbers less than 8, then the statement is true.
2. 8 is not an element of the given set, so the statement is false.
3. There is no negative natural number or counting number, so the statement is false.
4. Since ½ is not an element of integer, then the statement is true.

Basic Number Sets

The following sets of numbers are commonly used in mathematics.

Remember!

Natural Number or Counting Numbers N = { 1, 2, 3, 4, 5, ...}


Whole Numbers W = { 0, 1, 2, 3, 4, 5, ...}
Integers Z = { ..., -4, -3, -2, -1, 0,1, 2, 3, 4, 5, ...}
Rational Numbers Q = the set of all terminating or repeating decimals
Irrational Numbers Q’ = the set of all nonterminating, nonrepeating decimals
Real Numbers R = the set of all rational and irrational numbers

Set Description

In mathematics several ways the elements of a set can be described but for simplicity
of our discussion, we will be using the two methods: Roster and Rule Method.

1. Roster Method. Indicate a set by listing the elements separated by commas and
enclosing them in braces.

2. Rule Method. Describing the the set using words and enclosing them in braces.

Example 3: Use the roster and rule methods to represent the following sets:

1. The set of the days in a week.


2. The set of numbers divisible by 5 less than 50

Solution:

1. Roster Method: A = { Monday, Tuesday, Wednesday, Thursday, Friday,


Saturday, Sunday}
Rule Method: A = {x | x is a day in a week}

14
2. Roster Method: B = { 5, 10, 15, 20,25, 30, 35, 40, 45}
Rule Method: B = { x | x is a number divisible by 5 less than 50}

Note: The vertical bar is read “such that” and the statement 2 is read as,
B is the set of all x such that x is a number divisible by 5 less than 50.

Types of Sets

1. Finite set. A set is said to be a finite set if all possible elements can be enumerated
or listed down.

Example 4: The set of vowel letters in the English alphabet.


A = {a, e, i, o, u}

2. Infinite set. A set is said to be an infinite set if all possible elements are uncountable
or unlimited and cannot be listed down. Since not all elements can be enumerated,
ellipses (...) is used to indicate the continuity of the elements.
Example 5: The set of counting numbers greater than 20.
A = {21, 22, 23, 24, 25, 26, 27, 28, 29, 30, 31, ...}

3. Empty set. Also known as null set. A set that contains no elements. The symbol Ø or
{ } is used to represent the empty set.

Example 6: The set of counting numbers between 5 and 6.


A=Ø

4. Unit set. A set that contain only one element.

Example 7: A set of counting number less than 2.


A = {1}

5. Equivalent sets. Two sets A and B are said to be equivalent , denoted by A ~ B, if and
only if A and B have the same number of elements.

Example 8: A = { 1 , 2, 3, 4, 5} and B = {a , b, c, d, e}. Since sets A and b have the


same number of elements, then A ~ B.

6. Equal sets. Two sets A and B are said to be equal, denoted by A = B, if and only if A
and B have exactly the same elements.

Example 9: A = {l, o, v, e} and B = {e, l, o, v}. Since sets A and b have the same
elements, then A = B.

7. Joint sets. Two sets A and B are said to be joint set if they contain at least one element
in common.

15
Example 10: If A = {1, 4, 5, 6} and B = { 2, 4, 8, 9}, then the sets are joints sets since
they contain one common element, 4.

8. Disjoint sets. Two sets A and B are said to be disjoint set if they do not contain
common element.

Example 11: A = { 1, 2, 3, 7} and B = {4, 5, 9}. Set A and set B are disjoint sets since
there is no common element.
9. Subset. Given two sets A and B, if every element of set A is also an element of set B,
then A is called a subset of B and we write it as A ⊆ B.

Example 12. Let A = {2, 4, 8} and B = {2, 4, 6, 8, 10}. Since, all the elements of set A
are contained in set B, then A ⊆ B.

10. Super set. Whenever a set A is a subset of set B, we say that B is a superset of A and
we write, B ⊇ A.

Example 13. Let A = {2, 4, 8} and B = {2, 4, 6, 8, 10}. Here, A ⊆ B but B ⊇ A.

11. Proper subset. If A and B are two sets, then A is called the proper subset of B, denoted
by A ⊂ B, if A ⊆ B and B ⊇ A but A ≠ B.

Example 14. If A = {1, 2, 3, 4} and B = {1, 2, 3, 4, 5}, we observe that, all the elements
of A are present in B but the element ‘5’ of B is not present in A. So, we
say that A is a proper subset of B. Symbolically, we write it as A ⊂ B.

12. Power set. The collection of all subsets of set A is called the power set of A. It is
denoted by P(A). In P(A), every element is a set including an empty set of A. The
number of subsets can be determined by using the formula 2 n, where n refers to the
number of elements in a given set.

Example 15: If A = {a, b, c} then the number of subsets of A is


23 = 8. Then all subsets of a will be
P(A) = {Ø,{a}, {b}, {c}, {a, b}, {a, c}, {b, c}, {a, b, c}}

13. Universal Set. If the set has all the possible elements under consideration, then the
set is know as universal set. It is denoted by U.

Example 16. If A = {1, 2, 3}, B = { 3, 4, 5} and C = { 6, 7, 8}, then the


U = {1 , 2, 3, 4, 5, 6, 7, 8}

16
Operation of Sets

Sets can be combined in a number of different ways to produce another set. There are
operations that involved in a set. The following operations are union, intersection,
complementation, set difference, and cartesian product.

1. Union of Two Sets

The symbol U is employed to denote the union of two sets. Let us have two
sets A and B in symbol 𝐴 ∪ 𝐵 read “A union B” or “the union of set A and Set B” – is
defined as the set that consist of all elements belonging to either set A or set B (or
both).

A U B = {x | x ∈ A or x ∈ B}

Example 17: Find the union of the following sets:


1. A = {2, 3, 4} and B = { 3, 4, 5}
2. D = {1, 2, 3, 4, 5, 6, 7, 8, 9} and E = {5, 10, 15}

Solution:
1. A U B = {2, 3, 4, 5}
2. D U E = { 1, 2, 3, 4, 5, 6, 7, 8, 9, 10, 15}

Note: The elements of the union that are elements of both sets are listed
only once.

2. Intersection of two Sets

The intersection of the sets A and B, denoted by A ∩ B, read “A intersection B”


or “the intersection of set A and set B” is defined as the set of elements belongs to both
A and B, i.e. set of the common elements in A and B.

A Ո B = { x | x ∈ A and x ∈ B}

Example 18: Find the intersection of the following sets:


1. A = {1, 2, 3} and B = {1, 2, 4, 5}
2. C = {1, 3, 5} and D = {2, 4, 6}

Solution:
1. A Ո B = {1, 2}
2. C Ո D = Ø

Note: What happened in example 2? Since there are no common elements


to both two sets, then the intersection of the sets would be a null or empty
set.

17
3. Complement of a set

Let us consider set A. The complement of set A denoted by A’ read as “A


prime” is defined as the set whose elements are not in set A but elements of the
universal set U. In symbols,

A' = { x | x A but x ∈ U}

Example 19: Find the complement of set A = {2, 4, 6, 8} and U = {1, 2, 3, 4, 5, 6, 7, 8,


9, 10}

Solution:
A’ = {1, 3, 5, 7, 9, 10}

Example 20: Let us consider the following sets:


U = {1, 2, 3, 4, 5, 6, 7, 8, 9, 10}
A = {1, 3, 5, 7, 9}
B = {2, 4, 5, 8, 9}

Give the elements of:


a. A’
b. (A Ո B)´

Solution:
a. A’ = {2, 4, 6, 8, 10}
b. (A Ո B)´ = (A Ո B) = {5, 9}
(A Ո B)´ = {1, 2, 3, 4, 6, 7, 8, 10}
4. Set difference

Given set A and set B, the set difference of set B from set A is the set of all
elements in A, but not in B. In symbols:

A - B = { x | x ∈ A but x B}

Example 21: Find the difference.

1. Given set A = {a, b, c, d, e, f, g} and B = {a, c, f, h, k, u}, find 𝐴 − 𝐵.


2. Find 𝐵 − 𝐴 from the given sets below.
A = {x | x is a number greater than 6 and less than 10}
B = { x | x is a number less than 15}

Solution:
1. A – B = { b, d, e, g}
2. First write the sets in roster method to view the elements
A = { 7, 8, 9}
B = { 1, 2, 3, 4, 5, 6, 7, 8, 9, 10, 11, 12, 13, 14}
B – A = {1, 2, 3, 4, 5, 6, 10, 11, 12, 13, 14}

18
5. Cartesian Product

The cartesian product of sets A and B, denoted A × B, is the set of all possible
ordered pairs where the elements of A are first and the elements of B are second.

In set-builder notation, A × B = {(a, b) : a ∈ A and b ∈ B}.

Example 22. Find A x B, whose set 𝐴 = {𝐻, 𝑇} and 𝐵 = {1, 2, 3, 4, }

Solution: 𝐴 x B = {(H, 1), (H, 2), (H, 3), (H, 4), (T, 1), (T, 2), (T, 3), (T, 4)}

Example 23. A couple is planning their wedding. They have three nieces (Ann, Betty,
and Deanne) and two nephews (Ed, Fred). How many different pairings
are possible to have one boy and one girl as a ring bearer and flower girl?

Solution:
This problem can be considered as cartesian product.

Let A be the set of nieces A = {Ann, Betty, Deanne}


Let B be the set of nephews B = {Ed, Fred}
A x B = { (Ann, Ed), (Ann, Fred), (Betty, Ed), (Betty, Fred), (Deanne, Ed),
(Deanne, Fred)}
Thus, the number of pairs is 6.

Venn Diagram

A Venn Diagram is a pictorial representation of the relationships between two or more


sets. We can represent sets using Venn diagrams. In a Venn diagram, the sets are
represented by shapes; usually circles or ovals. The elements of a set are labeled within the
circle.
The following diagrams show some set operations and Venn diagrams for complement
of a set, disjoint sets, subsets, intersection and union of sets.

Set A A’ complement of A

A and B are disjoint sets B is a proper subset of A


19
U U

A intersect B A union B

The set of all elements being considered, the universal set is represented by a
rectangle.

Example 24. In a class of 80 students, each of the students passed either in Mathematics or
in English or in both. 50 students passed in both and 20 passed in English. Find
how many students passed in Mathematics?

Solution: Let "x" be the number of students passed in Mathematics. By representing the
given details in Venn diagram, we get

x 50 20

M E

From the Venn diagram:

x + 50 + 20 = 80
x = 80 - 70 = 10
x = 10

Number of students who passed in Mathematics

= x + 50 = 10 + 50 = 60

Therefore, 60 students passed in Mathematics.

Example 25. The population of a town is 5000. Out of these, 1500 individuals read newspaper
A and 2700 read newspaper B. 1000 individuals read both newspapers. Find
the number of individuals who do not read neither of the two papers.

20
Solution: Let: A = The set of individuals who read newspaper A
B = The set of individuals who read newspaper B

1500 2700
-1000 1000 -1000
500 1700
A B

Number of persons who read at least one newspaper

= 500 + 1000 + 1700


= 3200
To find the number of individuals who do not read neither of the two papers, we
have to subtract number of persons who read at least one from total population.

= 5000 - 3200
= 1800

So, the number of individuals who do not read neither of the two papers is 1800.

Example 26. In a school, all the students play either basketball or volleyball or both. 80
students play basketball, 75 students play volleyball and 60 students play both
games. Find
a. the number of students who play Basketball only.
b. the number of students who play Volleyball only.
c. the total number of students in the school.

Solution: Let B = The set of students who play basketball


V = The set of students who play volleyball

80 75
- 60 60 - 20
20 55
B V

a. The number of students who play basketball only is 20

b. The number of students who play volleyball only is 55

c. The total number of students in the school


= 20 + 55 + 60
= 135

21
Example 27. There are 100 students surveyed and asked which of the following subjects they
take this semester: Mathematics, English, or Biology. The result revealed that
35 responded English, 50 responded Mathematics, 29 responded Biology, 12
responded Mathematics and English, 8 responded English and Biology, 11
responded Biology and Math, and 5 responded all. Determine

a. How many students are not taking any of the three subjects?
b. How many students take Math, but not Biology or English?
c. How many students take Math and English, but not Biology?

Solution: In this problem, we have four regions which overlap. The easiest strategy is to start
at the center, the part where the three circles overlap. In short, we start from bottom
to the top in the result above.

1. Five students responded that they took all


the subjects, so we put 5 at the center.

2. Eleven responded Biology and Math. So,


we should put it in the Biology-Math
overlap. However, of the 11 who takes
Biology and Math, 5 were also
taking English as shown in (1). So, there
are 11-5 = 6 students in the Math-Biology
overlap.

3. Eight responded Biology and English. But of


those 8 taking Biology and English, 5 are also
taking Math. So, there are 8 – 5 = 3 students
who are taking Biology and English.

Also, there are 12 students who are taking


Math and English, and of those 12, 5 are also
taking Biology, so there are 12 – 5 = 7 who
are taking Math and English. Now see the
next figure to see how the Venn diagram
should look like after this step.

4. Next, 29 students responded Biology. But


notice that 6, 5, and 3 are already in the
Biology circle. So, we subtract those students
from 29. That is, 29 – (6 + 5 + 3) = 29 – 14 =
15. So, there are 15 students who take only
Biology. 22
5. Lastly, there are 50 students who are taking
Math and 35 who are taking English. But in
the Math circle there are 6 + 5 + 7 students
who are also taking the other subjects and in
the English circle there
are 3 + 5 + 7 students who are also taking
the other subjects. Therefore, we can have
the following calculations:

a.Number
To answer the firstwho
of students question, howMath
take only many = students
50 did not take any of the three subjects,
recall
– (6 + 5 that
+ 7) there
= 32 are 100 students who were surveyed. If we add all the numbers in the
diagram, 15 + 32 + 20 + 6 + 7 + 3 + 5, the sum is only 88. Therefore, 100 – 88 = 12
Number
studentsof Students whoany
did not take take
of only English
the three =
subjects.
35 – (3 + 5 + 7) = 20

b. The number of students taking Math, but not Biology nor English is 32.

c. The number of students who are taking Math and English is 7.

LESSON 2
THE LANGUAGE OF RELATIONS AND FUNCTIONS

Relation

When you group two or more points in a set, it is referred to as a relation. When you
want to show that a set of points is a relation, you list the points in ordered pairs. A relation is
just a set of ordered pairs. There is absolutely nothing special at all about the numbers that
are in relation. In other words, any bunch of numbers is a relation so long as these numbers
come in pairs.

Example 28: {(2, 4), (3, 5), (4, 8), (6, 7) usually written in set notation.

where: Range or inputs (x – values) = {2, 3, 4, 6}


Domain or outputs (y – values) = {4, 5, 8, 7}

23
You will notice in the above example that, relation shows the relationship between
INPUT and OUTPUT, merely the range and domain. There are other ways to write the relation
aside from set notation such as through tables, plotting in XY coordinates and mapping
diagram as shown below;

X Y
y
2 1 2 1

2 -2 2 -2
0 -3 x -3
0
1 4
1 4
3 1
3 1

Relation in table Relation in Graph Relation in mapping diagram

What is a Function?

Function relates an input to an output


Range or
Input
(independent variable)
𝒚 = 𝒇(𝒙)
output
Name of function
Domain or
(dependent variable)

A function is a relation when each element of the domain is paired to exactly one
element in the range. In other words, there is one and only one output (y) with each input (x).

Function can be illustrated as of the following;

1. Set of Order pairs

{(3, 5), (6, 8), (2, 4), (7, 1)} Function

{(4, 2), (3, 2), (5, 2), (6, 2)} Function

{(1, 2), (0, 3), (1, 6), (5, 4)} Not a Function

24
The first two examples are functions since for every one output there is one
corresponding input unlike the third example that there is two or more output to a
single input, therefore it is not a function. Note that, the first element (x – domain) is
the independent variable, while the second element (y – range) is the dependent
variable.

2. Mapping or arrow diagram

Examples:

3 a 3
4 b 4
5 5 8
c 9
Function One to One Relation Function Many to One Relation

3 3 1
4 4 7
9
5 5

Not function, One to Many Relations Not function, not all elements are represented

3. Graphing

Vertical line test

y y

x x

Function Function Function

Vertical line passes only once in the graph; therefore, it is a function.

25
Not Function Not Function

The vertical line passes through more than one point of the graph, therefore that
relation is not a function.

Evaluating a Function

When we have a function in formula form, it is usually a simple matter to evaluate the
function. For example, the function 𝑓(𝑥) = 5 − 3𝑥 2 can be evaluated by squaring the input
value, multiplying by 3, and then subtracting the product from 5.

How will you evaluate a function given its formula?

Follow this simple step:

1. Replace the input variable in the formula with the value provided.
2. Calculate the result.

Example 29: Evaluate the function 𝑓(𝑥) = 𝑥 2 + 2𝑥 when x = 4

Solution:

𝑓(4) = (4)2 + 2(4) replacing the input in the formula

𝑓(4) = 16 + 8 calculating the result

𝑓(4) = 24 answer

26
Module3:
Problem Solving

Introduction
M ost occupations require workers to have good problem-solving skills . For instance,
engineers and architects must solve many complicated problems as they design and construct
modern buildings that are aesthetically functional, pleasing, and that meet stringent safety
requirements. Two goals of this chapter are to help you become a better problem solver and
to demonstrate that problem solving can be an enjoyable experience.

Learning Outcomes:
At the end of this chapter, you are expected to:

 Differentiate inductive reasoning from deductive reasoning.


 Use different types of reasoning to justify statements and arguments made
about mathematics and mathematical concepts.
 Solve problems using inductive and deductive reasoning.
 Solve problems involving patterns and recreational problems.
 Solve problems employing Polya’s four steps.
 Increase awareness on the importance of reasoning and problem solving

LESSON 1:
INDUCTIVE AND DEDUCTIVE REASONING

Inductive Reasoning

Inductive and deductive reasoning are considered the two fundamental forms of
reasoning for mathematicians. Inductive reasoning is a process that makes use of our
knowledge in making a general inference about unfamiliar occurrences based on observation
and patterns. For example, students use this type of reasoning when they look at many
different quadrilaterals, and try to list the characteristics they have in common. The reasoning
process is enhanced by considering figures that are not quadrilaterals and discussing how
they are different.
An example of inductive reasoning is, for example, when you notice that all the dogs
you see around are black and white so you make the conclusion that all dogs in the world are
black and white. Can you say for certain that this conclusion is correct? No, because it is based

27
on just a few observations. However, it is the beginning of forming a correct conclusion, or a
correct proof.This observation has given you a starting hypothesis to test out.

Remember!
Inductive reasoning is the process of reaching a general conclusion
by examining specific examples.

By examinining a list of numbers and predict the next number in the list according to
some pattern you have observed, you are using inductive reasoning.

Example 1. Use Inductive Reasoning to Predict a Number

Use inductive reasoning to predict the next number in each of the following list.

a. 4, 8, 12, 16, 20, _____ b. 2, 4, 8, 16, 32, _____

Solution:

a. Each successive number is 4 greater than the preceding number. Thus, we predict
that the next number in the list is 4 greater than 20, which is 24.
b. Each preceding number is doubled to get the succeeding number. The number 16
is obtained by multiplying 8 by 2. The number 32 is obtained by mutltiplying 16 by
2. Since the last number is 32, we predict that that next number is 64. It is obtained
by mutlpying 32 by 2.

Inductive reasoning is not used just to predict the next number in a list. In the
succeeding example, we use inductive reasoning to make a conjecture about an arithmetic
procedure.

Example 2. Use Inductive Reasoning to Make a Conjecture

Consider the following procedure: Pick a number. Multiply the number by 9, add
15 to the product, divide the sum by 3, and subtract 5.
Complete the above procedure for several different numbers. Use inductive
reasoning to make a conjecture about the relationship between the size of the
resulting number and the size of the original number.

Solution:
Suppose we pick 5 as our original number. Then the original number would
produce the following results:
Original number: 5
Multiply by 9: 9 x 5 = 45
Add 15: 45 + 15 = 60
Divide by 3: 60 ÷ 3 = 20
Subtract 5: 20 – 5 = 15

28
We started with 5 and followed the procedure to produce 15. Starting with 6 as our
original number produces a final result of 18. Starting with 10 produces a final result of 30.
Starting with 100 produces a final result of 300. In each of these cases, the resulting number
is three times the original number. We conjecture that following the given procedure produces
a number that is three times the original number.

Counterexamples

A statement is a true statement provided that it is true in all cases. If you can find one
case for which a statement is not true, called a counterexample, then the statement is a false
statement. In Example 3, we verify that each statement is a false statement by finding a
counterexample for each.

Example 3. Find a Counterexample

Verify that each of the following statements is a false statement by finding a


counterexample.

For all numbers x:

a. |x| > 0 b. x2 > x

Solution:

A statement may have many counterexamples, but we need only to find one
counterexample to verify that the statement is false.

a. Let x = 0. Then /0/ = 0. Because 0 is not greater than 0, we have found a


counterexample. Thus, “for all numbers x, /x/ > 0” is a false statement.
b. For x = 1, we have 12 = 1. Since 1 is not greater than 1, we have found a
counterexample. Thus, “for all numbers x, x2 > x” is a false statement.

QUESTION: How many counterexample is needed to prove that a statement is


false?

Deductive Reasoning

Another type of reasoning is called deductive reasoning. Deductive reasoning is


distinguished from inductive reasoning that it is the process if reaching a conclusion by
applying general principles and procedures. Deductive reasoning involves making a logical
argument, drawing conclusions, and applying generalizations to specific situations. For
example, once students have developed an understanding of “triangle”, they apply that
generalization of new figures to decide whether or not each is a triangle. The conclusions
reached by this type of reasoning are valid and can be relied on.

29
Remember!
Deductive reasoning is the process of reaching a conclusion by
applying general assumptions, procedures or principles.

Illustrative Examples
1. If a number is divisible by 2, then it must be even.
12 is divisible by 2.
Therefore, 12 is an even number.

2. All Mathematics teachers know how to play Sudoku.


Resty is a Math teacher.
Therefore, Resty knows how to play Sudoku.

3. If a student is a DOST scholar, he receives a monthly allowance.


If a student receives a monthly scholar, his parents will be happy.
Therefore, if a student is a DOST scholar, his parents will be happy.

4. If ∠A and ∠ B are supplementary angles, their sum is 180°.


If m⦞∠A = 100°, then m∠ B = 80°.

Example 4. Use Deductive Reasoning to Make a Conjecture

Use deductive reasoning to show that the following procedure will a number that
is four times the original number.

Procedure: Pick a number. Multiply the number by 8, add 6 to the product, divide
the sum by 2, and subtract 3.

Solution:
Let n represents the original number.

Multiply the number by 8: 8n


Add 6 to the product: 8n + 6
Divide the sum by 2: (8n + 6) / 2 = 4n + 3
Subtract 3: 4n + 3 – 3 = 4n

Inductive Reasoning vs Deductive Reasoning

In example 5, we will analyze arguments to determine whether they used inductive


reasoning or deductive reasoning.

30
Example 5. Determine Types of Reasoning

Determine whether each of the following arguments is an example of inductive


reasoning or deductive reasoning.

a. During the past 10 years, a tree has produced plums every other year. Last
year, the tree did not produce plums, so this year the tree will produce plums
b. All home improvement cost more than the estimate. The contractor estimated
that my home improvement will cost P70,000. Thus, my home improvement
will cost more than P70,000.

Solution:

a. This argument reaches a conclusion based on specific examples, so it is an


example of inductive reasoning.
b. Because the conclusion is a specific case of a general assumption, this argument
is an example of deductive reasoning.

Logic Puzzles

Logic puzzles can be solved by using deductive reasoning and a chart that enables us
to display the given information in visual manner.

Example 6. Solve a Logic Puzzle

Each of four neighbors, Flor, Lyn, Erl, and Cha, has a different occupation
(teacher, banker, chef, or broker). From the following clues, determine the
occupation of each neighbor.

1. Lyn gets home from work after the banker but before the broker.
2. Erl, who is the last to get home from work, is not the teacher.
3. The broker and Erl leave for work at the same time.
4. The banker lives next door to Cha.

Solution:
From clue 1, Lyn is neither the banker nor the broker. In the following chart,
write X1 (which stands for “ruled out by clue 1”) in the Banker and the Broker columns
of Flor’s row

Teacher Banker Chef Broker


Flor
Lyn X1 X1
ERL
Cha

31
From clue 2, Erl is not the teacher. Write X2 (ruled out by clue 2) in the teacher
column of Erl’s row. We know from clue 1 that the banker is not the last to get home,
and we know from clue 2 that Linda is the last to get home; therefore, Lyn is not the
banker. Write X2 in the Banker column of Erl’s row.

Teacher Banker Chef Broker


Flor
Lyn X1 X1
Erl X2 X2
Cha

From clue 3, Erl is not the dentist. Write X3 for this condition. There are now Xs
for three of the four occupations in Erl’s row; therefore Erl must be the Chef. Put a /
(check mark) in that box. Since Erl is the chef, none of the other three people can be
the chef. Write X3 for these conditions. There are now Xs for three of the four
occupations in Lyn’s row; therefore, Lyn must be the teacher. Insert a / (check mark)
to indicate that Lyn is the teacher, and write X3 twice to indicate that neither Flor nor
Cha is the teacher.

Teacher Banker Chef Broker


Flor X3 X3
Lyn / X1 X3 X1
Erl X2 X2 / X3
Cha X3 X3

From clue 4, Cha is not the banker. Write X4 for this condition. See the following
table. Since there are three Xs in the Banker column, Flor must be the Banker. Put a /
in that box. Thus Flor cannot be the broker. Write X4 in that box. Since there are Xs in
the broker column, Cha must be the broker. Put a / in that box.

Teacher Banker Chef Broker


Flor X3 / X3 X4
Lyn / X1 X3 X1
Erl X2 X2 / X3
Cha X3 X4 X3 /

Flor is the banker, Lyn is the teacher, Erl is the chef, and Cha is the broker.

LESSON 2:
KENKEN PUZZLE

KenKen is an arithmetic-based logic puzzle invented by Tetsuya Miyamoto, a


Japanese mathematics teacher, in 2004. The noun “kenken” has “knowledge” and
“awareness” as their meanings. Hence, Kenken translates as knowledge squared, or
awareness squared.

32
In the previous years, the KenKen’s popularity has increased at a dramatic rate. More
than a million of KenKen puzzle books have been sold, and KenKen puzzles now appear in
many famous nerwspapers, including the New York Times and the Boston Globe.
Similar to Sudoku puzzles, KenKen puzzles also require you to perform arithmetic to
solve the puzzle.

Remember!

Rules for Solving KenKen Puzzle

1) For a 3 by 3 puzzle, fill in each box (square) of the grid with one of the
numbers 1, 2, or 3.
2) For a 4 by 4 puzzle, fill in each square of the grid with one of the numbers 1,
2, 3, or 4.
3) For a n by n puzzle, fill in each square of the grid with one of the numbers 1,
2, 3, …, n.

Grids range in size from a 3 by 3 up to a 9 by 9.


 A number in any row or column must not be repeated
 The numbers in each heavily outlined set of squares (cages), must
combine (in some order) to produce the target number in the top left
corner of the cage using the mathematical operation indicated.
 Cages with only one square should be filled in with the target number.
 A number may be repeated within a cage as long as it is not in the same
row or column.

Following is a is a 4 by 4 puzzle with its solution. Puzzles that are constructed properly
have a unique solution.

33
Basic Puzzle Solution Strategies

Single-Square Cages. Fill cages that consist of a single square with the target number for
that square.

Cages with Two Squares. Examine the cages with exactly two squares. Many cages that
cover two squares will just have two digits that can be used to fill the cage. For instance, in a
5 by 5 puzzle, a 10X cage with exactly two squares can only be filled with 2 and 5 or 5 and 2.

Large or Small Target Numbers. Search for cages that have an unusually large or small
target number. These cages generally have only a few combinations of numbers that can be
used to fill the cage.

Examples:
In a 6 by 6 puzzle, a 120x cage with exactly three squares can only be filled with 4, 5,
and 6.

A 3+ cage with exactly two squares can only be filled with 1 and 2.

Duplicate Digit in a Cage. Consider a 3x cage. The digits 1, 1, and 3 produce a product of 3;
however, we cannot place the two 1s in the same row or the same column. Thus the only way
to fill the squares is to place the 3 in the corner of the L-shaped caged as shown below.
Remember: A digit can occur more than once in a cage, provided that it does not appear in
the same row or in the same column.

Remember the Following Rules

In an n by n puzzle, each row and column must contain every digit from 1 to n.

In a 2-square cage involving subtraction or division, the order of the numbers in the
cage is not important. For instance, a 3- cage with two squares could be filled with 1 and 4 or
with 4 and 1. A 3÷ cage with two squares could be filled with 1 and 3 or with 3 and 1.

34
LESSON 3
POLYA’S PROBLEM SOLVING STRATEGY

The National Council of Teachers of Mathematics (NCTM) suggests that problem


solving plays an important role in mathematics and should have a popular role in the
mathematics of K-12 students. The term “problem solving” refers to mathematical tasks that
have potential to provide intellectual challenges for enhancing students’ mathematical
understanding and development.

One of the foremost recent mathematicians to make a study of problem solving was
George Polya (1887-1985). He was born in Hungary and moved to the United States in 1940.
The basic problem solving strategy that Polya advocated consisted of the following four steps.

Remember!
Polya’s Four-step Problem Solving Strategy
1. Understand the Problem
2. Devise a Plan
3. Carry out the plan.
4. Review the solution.

Understand the Problem. Before anything else, you must understand the problem. Have a
full grasp of what are known and unknown. To help you understand the problem, consider the
following.
 Restate the problem using your own words
 List what is known about these types of problems
 Determine missing information that, if known, would allow you to solve the problem
 Identify the goal

Devise a Plan. Apply strategies. To devise a plan requires skill and own judgment. Some
strategies are as follows .
 Make a list of the known information.
 Make a list of information that is needed.
 Draw a diagram.
 Make an organized list that shows all the possibilities.
 Make a table or a chart.
 Work backwards.
 Try to solve a similar but simpler problem.
 Look for a pattern.
 Write an equation. If necessary, define what each variable represents.
 Perform an experiment
 Guess at a solution and then check your result.

35
Carry Out the Plan. After devising a plan, you must carry it out.
 Implement the stragegy in Step 2 and perform anh necessary actions or computations.
 Check each step of the plan as you proceed; this may be intuitive checking or a formal
proof of each step.
 Keep an accurate record of your steps as you implement your devised plan.

Look back. Once you have found a solution, look back at what you have done.
 Ensure that the solution is consistent with the facts of the problem.
 Interpret the solution in the context of the problem.
 Ask yourself whether there are generalizations of the solution that could apply to other
problems.

Example 7. Apply Polya’s Strategy (Make an organized list)

A baseball team won two out of their fast four games. In how many different orders
could they have two wins and two losses in four games?

Solution.

Understand the Problem

There are many different orders. They team may have won two straight games and
lost the last two (WWLL). Or maybe they lost the first two games and won the last two
(LLWW). Of course there are other possibilities, such as WLWL.

Devise a Plan

We will make an organized list of all the possible orders. An organized list is a list that
is produced using a system that ensures that each of the different orders will be listed
once and only once.

Carry Out the Plan

Each entry in our list must contain two Ws and two Ls. We will use a strategy that makes
sure each order is considered, with no duplications. Once such strategy is to always
write a W unless doing so will produce to many. Ws or a duplicate of one of the previous
orders. If it is not possible to write a W, then and only then do we write an L. This strategy
produces the six different orders shown below.
1. WWLL (Start with two wins)
2. WLWL (Start with one win)
3. WLLW
4. LWWL (Start with one loss)
5. LWLW
6. LLWW (Start with two losses)

Look back

36
We have made an organized list. The list has no duplicates and the list considers all
possibilities, so we are confident that there are six different orders in which a baseball
team can win exactly two out of four games

Module 4:
Data Management
Introduction
The role of data management tools is important to further analyze and interpret data.
Utilizing these tools will greatly enhance the theories that might be otherwise misunderstood.
This module deals with the computation of the following measures as follows:
a) central tendency, b) dispersion, c) relative position, and d)normal distribution.

Learning Outcomes:
At the end of this chapter, you are expected to:

 Utilize various data management tools to practice and achieve quantitative


data;
 Calculate the measure of central tendency, measures of dispersion, and
measures of relative position, and standard score of a given set of data.
 Interpret data based on the result of computation.
 Appreciate the importance and application of measures of central tendency,
measures of dispersion, measures of relative position, and normal
distribution in real life situations.

LESSON 1:
MEASURES OF CENTRAL TENDENCY

A measure of central tendency is a summary statistic that represents the center point
or typical value of a dataset. It also referred to as the central location of a distribution. There
are three measures of central tendency - mean, median, and mode. It depends on the type of
data as to choose the most suitable measure of central tendency.

37
Mean for Ungrouped Data

The most commonly used measure of central position is the mean (also known as
arithmetic mean). It is obtained by dividing the total measures by the number of measures in
a variable. It is symbolized as 𝑋̅ (read as x bar). Mean is appropriate to use when the
distribution is at least interval scale.
To find the mean of ungrouped data, use the formula

Remember!
Ʃ𝑥
𝑥̅ =
𝑛
where:
∑ 𝑥 = sum of entries 𝑛 = number of entries

Example 1: The scores of a 100-item test in an Algebra class of 12 students are 89, 86, 87,87,
88, 92, 81, 84, 80, 78, 80, 82. Find the mean scores of the 12 students.

Solution:

89+86+87+87+88+92+81+84+80+78+80+82
𝑥̅ = = 84.5
12

Weighted Mean

Occasionally, we want to find the mean of a set of values wherein each value or
measurement has a different weight or degree of importance. We call this the weighted mean
and the formula for computing it is as follows:

Remember!
Ʃ𝑥𝑊
𝑥̅ =
∑𝑊
where:
𝑥 = measurement or value 𝑊 = weight

Example 2: Below are Maria’s subjects and the corresponding number of units and grades
she got for the first grading period. Compute her grade point average.

Subject No. of Units (w) Grade (x)


Math 1.5 80
English 1.5 82
Filipino 1 83
Science 2 81
Social Studies 1 80
PEHM 1.5 85
THE 2 82

38
Solution:

x
 xW
W
80(1.5)  82(1.5)  83(1)  81(2)  80(1)  85(1.5)  82(2)

10.5

120  123  83  162  80  127.5  164



10.5

859.5

10.5

x  81.86

Therefore, Maria has the GPA of 81.86 for the first grading period.

Mean for Grouped Data

To compute the mean of grouped data, employ the formula below. This is applicable
when the data is huge, thus grouping of items is most convenient to apply.

Remember!
Ʃ𝑓𝑥
𝑥̅ =
Ʃ𝑓
where:
x = class mark (class interval midpoint)
f= class frequency

Example 3: Compute the mean of the scores of the students in a Mathematics test.

Class Interval Frequency

40 – 44 6
35 – 39 12
30 – 34 13
25 – 29 12
20 – 24 6
15 – 19 3
10 -14 2

39
Solution: Here is the frequency distribution of the given data. Get the sum of the columns x
and fx.

Class Interval f x fx

40 – 44 6 42 252
35 – 39 12 37 444
30 – 34 13 32 416
25 – 29 12 27 324
20 – 24 6 22 132
15 – 19 3 17 51
10 - 14 2 12 24
Ʃ𝑓 = 54 Ʃ𝑓𝑥 = 1,643

Ʃ𝑓𝑥
𝑥̅ =
𝑛

1,643
𝑥̅ =
54

𝑥̅ = 30.43

The mean of the data is 30.43

Example 4: Compute for the mean gross sale of Mary’s Drygoods for half year.

Sales in Pesos Frequency


3,501 - 4,000 4
3,001 - 3,500 5
2,501 - 3,000 7
2,001 - 2,500 6
1,501 - 2,000 8
1,001 - 1,500 4
501- 1,000 2
1- 500 2

Solution: The frequency distribution for the data is given below. The columns x and fx are
added.

Sales in Pesos f x fx
3,501 - 4,000 4 3,750.5 15002.0
3,001 - 3,500 5 3,250.5 16252.5
2,501 - 3,000 7 2,750.5 19253.5
2,001 - 2,500 6 2,250.5 13503.0
1,501 - 2,000 8 1,750.5 14004.0
1,001 - 1,500 4 1,250.5 5002.0
501 - 1,000 2 750.5 1501.0
1 - 500 2 250.5 501.0
Ʃ𝑓 = 38 Ʃ𝑓𝑥 = 85019

40
Ʃ𝑓𝑥
𝑥̅ =
𝑛

85019
𝑥̅ =
38

𝑥̅ = 2237

The mean gross sale is Php 2,237.

Median for Ungrouped Data

The median is the middlemost item in a set of data, arranged from highest to lowest
or vice-versa. It is not affected by the individual measures of data set nor affected by the size
of extreme values. This is appropriate to use when the distribution is at least ordinal scale
since ranking of the data is involved.

How to compute the median of a given set of data:

1. Organize the data in either increasing or decreasing order.


2. Get the middlemost value. When the number of entries is odd, the middlemost value
is the median. When the number of entries is even, take the average of the two
middle measures.

Example 5: The number of chairs borrowed in the barangay from weekdays last week were
60, 62, 56, 37, and 99 respectively. Compute for the median.

Solution: Arrange the entries in increasing order.

37, 56, 60, 62, 99

The median is 60.

Example 6: Joy’s quizzes for the third quarter are 9, 8, 7, 11, 10, 6, 10, 7, 11, and 8. Solve for
the median.

Solution: Organize the entries in increasing order.

6, 7, 7, 8, 8, 9, 10, 10, 11, 11

Since the number of entries is even, compute the average of the two middle scores.

8+9
Md = = 8.5
2

41
Median of Grouped Data

To compute for the median of grouped data, identify first the median class, the class
interval holding the median. Since the median divides the distribution into two equal parts, first
get 50% of the total number of cases or scores. Then identify the interval containing the score
where 50% of the cases would fall below this value.

In computing for the median of grouped data, the following formula is used:

Remember!
Ʃ𝑓
− 𝑐𝑓
𝑀𝑑 = 𝑙𝑏𝑚𝑐 + ( 2 )𝑖
𝑓𝑚𝑐
where:
𝑙𝑏𝑚𝑐 = true lower limit or lower-class boundary of the median class
cf = cumulative frequency of the lower class next to the median class
𝑓𝑚𝑐 = frequency of the median class
f = frequency of each class;
i = class size

Ʃ𝑓
The median class is the class that contains the 𝑡ℎ quantity. The computed median
2
must be within the median class.

Example 7: Compute the median of the scores of the students in a Mathematics test.

Class Interval Frequency

40 – 44 6
35 – 39 12
30 – 34 13
25 – 29 12
20 – 24 6
15 – 19 3
10 -14 2

Solution: The frequency distribution for the data is given below. The columns for lb and “less
than” cumulative frequency are added.

Class Interval f lb <cf

40 - 44 6 39.5 54
35 – 39 12 34.5 48
30 – 34 13 29.5 36
25 – 29 12 24.5 23
20 – 24 6 19.5 11
15 – 19 3 14.5 5
10 - 14 2 9.5 2
Ʃ𝑓 = 54

42
Ʃ𝑓 54
Since = = 27, the 27th quantity is in the class 30 - 34. Hence, the median class
2 2
is 30 - 34.
Ʃ𝑓
− 𝑐𝑓
2
𝑀𝑑 = 𝑙𝑏𝑚𝑐 + ( )𝑖
𝑓𝑚𝑐

54
− 23
𝑀𝑑 = 29.5 + ( 2 )5
13

𝑀𝑑 = 29.5 + 1.54

𝑀𝑑 = 31.04

Therefore, median = 31.04.

Example 8: Solve for the median gross sale of Jane’s Drugstore for one month.

Sales in Pesos Frequency


3,501 - 4,000 4
3,001 - 3,500 5
2,501 - 3,000 7
2,001 - 2,500 6
1,501 - 2,000 8
1,001 - 1,500 4
501 - 1,000 2
1 - 500 2

Solution: The frequency distribution for the data is given below. The columns for lb and “less
than” cumulative frequency are added.

Sales in Pesos f lb <cf


3,501 - 4,000 4 3,750.5 38
3,001 - 3,500 5 3,250.5 24
2,501 - 3,000 7 2,750.5 29
2,001 - 2,500 6 2,250.5 22
1,501 - 2,000 8 1,750.5 16
1,001 - 1,500 4 1,250.5 8
501 - 1,000 2 750.5 4
1 - 500 2 250.5 2
Ʃ𝑓 = 30

Ʃ𝑓 38
Since = = 19, the 19th quantity is in the class 2, 501- 3,000. Hence, the median
2 2
class is 2501- 3,000.
Ʃ𝑓
− 𝑐𝑓
𝑀𝑑 = 𝑙𝑏𝑚𝑐 + ( 2 )
𝑓𝑚𝑐

43
38
− 16
𝑀𝑑 = 2000.5 + ( 2 ) 500
6

𝑀𝑑 = 2000.5 + 250

𝑀𝑑 = 2,250.5

The median gross sale = Php 2,250.5

Mode for Ungrouped Data

Another measure of position is the mode. It is the most occurring entry in a data set
and it’s the value having the highest frequency. Mode is appropriate to use when the variable
measured is in the nominal scale.

To find the mode for a set of data:

1. choose measure that appears most often in the set;


2. if two or more measures appear the same number of items, and the frequency they
appear is greater than any of other measures, then each of these values is a mode;
3. if every measure appears the same number of items, then the set of data has no
mode.

Example 9: The slippers size of 10 selected girls in a group are 7, 6, 5, 7, 5.5, 6, 7, 8, 8 and
7. What is the mode?

Solution: The mode is 7 since it is the slippers size that is with highest occurrence.

Example 10: The sizes of 9 groups in a certain institution are 52, 54, 57, 52, 53, 56, 57, 55
and 56.

Solution: The modes are 56 and 57 since the two measures occurred the same number of
times. The distribution is bimodal.

Mode for Grouped Data

To find mode of grouped data use the formula:

Remember!

𝐷1
where: 𝑀𝑜 = 𝐿𝑏𝑚𝑜 + ( )𝑖
𝐷1 + 𝐷2
LLmo = true lower limit or lower-class boundary of the modal class
D1 = is the difference between the frequencies of the modal class and
the next lower class
D2 = is the difference between the frequencies of the modal class and
the next upper class
i = the class size
44
The modal class is the class with the highest frequency. If binomial classes exist, any
of these classes may be considered as modal class.

Examples 11: Compute the mode of the scores of the students in a Mathematics test.

Class Interval Frequency


46 - 50 2
41 - 45 6
36 - 40 12
31 - 35 13
26 - 30 12
21 - 25 6
16 - 20 3
11 -15 2

Solution: The frequency distribution for the data given below. The column for lb is added.

Class Interval f lb
46 - 50 2 45.5
41 - 45 6 40.5
36 - 40 12 35.5
31 - 35 13 30.5
26 - 30 12 25.5
21 - 25 6 20.5
16 - 20 3 15.5
11 - 15 2 10.5

Since class 31 - 35 has the highest frequency, the modal class is 31 - 35.

𝐷1
𝑀𝑜 = 𝐿𝑏𝑚𝑜 + ( )𝑖
𝐷1 + 𝐷2

1
𝑀𝑜 = 30.5 + ( )5
1+1

𝑀𝑜 = 30.5 + 2.5

𝑀𝑜 = 33

The mode = 33

Example 12. Compute for the modal gross sale of Aling Cora’s Sari-sari Store for two months.

Sales in Pesos Frequency


3,501 - 4,000 4

45
3,001 - 3,500 5
2,501 - 3,000 7
2,001 - 2,500 6
1,501 - 2,000 8
1,001 - 1,500 4
501 - 1,000 2
1 - 500 2

Solution: The frequency distribution for the data is given below. The columns for lb is added.

Sales in Pesos f lb
3,501 - 4,000 4 3,500.5
3,001 - 3,500 5 3,000.5
2,501 - 3,000 7 2,500.5
2,001 - 2,500 6 2,000.5
1,501 - 2,000 8 1,500.5
1,001 - 1,500 4 1,000.5
501 - 1,000 2 500.5
1 - 500 2 0.5

Since the class 1,501 - 2,000 has the highest frequency, the modal class is 1,501 -
2,000.

𝐷1
𝑀𝑜 = 𝐿𝑏𝑚𝑜 + ( )𝑖
𝐷1 + 𝐷2

4
𝑀𝑜 = 1,500.5 + ( ) 500
4+2

𝑀𝑜 = 1,500.5 + 333.33

𝑀𝑜 = 1,833.33

The modal gross sale is Php 1,833.33.

LESSON 2:
MEASURES OF DISPERSION

The measures that describe the degree of extent of the data are called “measure of
dispersion” or “measure of variability” or “measure of spread”. This measure is used to
determine how scattered the values are in the distribution. In this topic, we will consider four
measures of dispersion, namely: range, average deviation, variance, and standard deviation.

46
Range for Ungrouped Data

The range is the simplest measure of variability. It is the difference between the largest
ad smallest measurement. To determine the range of ungrouped data, the formula is

Remember!
𝑅 = 𝐻 − 𝐿
where:
𝐻 = Highest measure L= Lowest measure

The main advantage of the range is that it does not consider every measure in the
data.

Example 13: Consider the four data sets presented below. Find the range of each data set.

Data Set
Data Set 1 11 12 13 14 15
Data Set 2 13 14 15 17 19
Data Set 3 10 15 18 20 22
Data Set 4 21 23 25 27 30
Solution:

Data Set 1: 𝑅 = 𝐻– 𝐿 Data Set 2: 𝑅 = 𝐻– 𝐿


` 𝑅 = 15 – 11 𝑅 = 19 – 13
𝑅 = 4 𝑅 = 6

Data Set 3: 𝑅 = 𝐻– 𝐿 Data Set 4: 𝑅 = 𝐻– 𝐿


` 𝑅 = 22 – 10 𝑅 = 30 – 21
𝑅 = 12 𝑅 = 9

Comparing the data sets, Data Set 1 has the least variation because it has the smallest
value of R. On the other hand, Data Set 3 has the most variation because it has the
largest value of R.
Range for Grouped Data

The range of a grouped data is simply the difference between the upper class boundary
of the top interval and lower class boundary of the bottom interval.

Example 14: Find the range of the scores in Midterm Exam of BEED First Year Students.

Class Interval Frequency


46 - 50 1
41 - 45 10
36 - 40 10
31 - 35 16
26 - 30 9
21 - 25 4

47
Solution:
Upper class boundary (UCB) = 50.5
Lower class boundary (LCB) = 20.5

UCB - LCB = 50.5 - 20.5 = 30

Average Deviation for Ungrouped Data

The Average Deviation (AD) is a measure of absolute dispersion that is affected by


every individual score. It is the mean of the absolute deviations of the individual scores from
the mean of all the scores.
A large average deviation would mean that a set of scores is widely dispersed about
the mean, while a small average deviation would imply that the set of scores is closer to the
mean.
The formula of average deviation for ungrouped data is:

Remember!
∑ |𝑥 − 𝑥̅ |
𝐴𝐷 =
𝑛 − 1

where: Ʃ = symbol for summation n = total number of scores


x = individual score 𝑥̅ = mean of all scores

To be able to apply the formula, the following steps can be observed:

1. Compute the mean from the given scores.


2. Subtract the mean from the individual scores to get the deviation. That is, 𝑥 − 𝑥̅
3. Get the absolute value of each deviation.
4. Get the sum of the absolute deviation and divide it by (n-1), where n is the total
number of scores. The quotient is the average deviation.

Example 15. The raw scores of eight students in Statistics are given as follows: 17, 17, 26,
28, 30, 30, 31, and 37. Compute the average deviation.

Solution:

Score Deviation Absolute Deviation


𝒙 𝒙 − 𝒙̅ |𝒙 − 𝒙 ̅|
17 -10 10
17 -10 10
26 -1 1
28 1 1
30 3 3
30 3 3
31 4 4
37 10 10
𝑥̅ = 27 Ʃ| 𝑥 − 𝑥̅ | = 42

48
∑ |𝑥 − 𝑥̅ |
𝐴𝐷 =
𝑛 − 1
42
𝐴𝐷 =
8−1
42
𝐴𝐷 =
7

𝐴𝐷 = 6

Example 16. The scores of nine students in Psychology are given as follows: 15, 19, 20, 24,
28, 30, 32, 32, and 40. Calculate the average deviation.

Solution:

Score Deviation Absolute Deviation


𝒙 𝒙 − 𝒙 ̅ |𝒙 − 𝒙 ̅|
15 -11.67 11.67
19 -7.67 7.67
20 -6.67 6.67
24 -2.67 2.67
28 1.33 1.33
30 3.33 3.33
32 5.33 5.33
32 5.33 5.33
40 13.33 13.33
𝑥̅ = 26.27 Ʃ| 𝑥 − 𝑥̅ | = 57.33

∑ |𝑥 − 𝑥̅ |
𝐴𝐷 =
𝑛 − 1
57.33
𝐴𝐷 =
9−1
57.33
𝐴𝐷 =
8

𝐴𝐷 = 7.17

The computed average deviation (A.D.) of scores in Statistics is 6 while test scores in
Psychology is 7.17. This can be interpreted as the scores in Statistics are less dispersed or
closely distributed near the mean (homogeneous) while the scores in Psychology are more
dispersed away from the mean (heterogeneous).

Average Deviation for Grouped Data

For the grouped data or scores organized in the form of frequency distribution, the
average deviation is computed as follows:

49
Remember!
∑ 𝑓𝑖 |𝑥𝑖 − 𝑥̅ |
𝐴𝐷 =
𝑛 − 1

where: Ʃ = symbol for summation n = total number of scores


xi = midpoint of the ith class interval 𝑥̅ = mean of all scores
fi = frequency of the ith class interval

The steps in determining the average deviation of grouped data are as follows:

1. Find the mean of the frequency distribution.


2. Get the midpoint of each class interval.
3. Subtract the mean from the midpoint of each class interval to get the deviations and
then, take their absolute values.
4. Multiply the frequency of each class interval to the corresponding absolute deviation
and add all the products to get ∑ 𝑓𝑖 |𝑥𝑖 − 𝑥̅ |.
5. Divide the sum by (n - 1), where n is the total number of frequencies. The quotient
is the average deviation.

Example 17. Below is a frequency distribution of test scores of 50 students in Assessment of


Learning. Find the average deviation.

Class Interval Frequency


36 – 40 7
31 – 35 10
26 – 30 5
21 – 25 14
16 – 20 6
11 – 15 8
Ʃ𝑓 = 50

Solution: The mean of this distribution is 𝑥̅ = 25.40.

Class ̅ ̅
𝒇𝒊 𝒙𝒊 ̅
𝒙𝒊 − 𝒙 |𝒙𝒊 − 𝒙| 𝒇𝒊 |𝒙𝒊 − 𝒙|
Interval
36 – 40 7 38 12.60 12.60 88.2
31 – 35 10 33 7.60 7.60 76
26 – 30 5 28 2.60 2.60 13
21 – 25 14 23 -2.40 2.40 33.6
16 – 20 6 18 -7.40 7.40 44.4
11 – 15 8 13 -12.40 12.40 99.2
𝑥̅ = 25.40 𝑛 = 50 ̅
Ʃ𝑓 𝑖 |𝑥𝑖 − 𝑥|
= 354.4

50
∑ 𝑓𝑖 |𝑥𝑖 − 𝑥̅ |
𝐴𝐷 =
𝑛 − 1
354.4
𝐴𝐷 =
50 − 1
354.4
𝐴𝐷 =
49

𝐴𝐷 = 7.23

We say that the scores deviate from the mean of 25.40 by an average of 7.23 units.

Variance for Ungrouped Data

Another way to avoid a sum of zero for the deviation scores is to square each deviation
score and get the average of all squared deviation scores. The resulting measure is called
“variance” which has a squared unit. In symbol, 𝑠2 .

To compute the variance of ungrouped data, the following formula may be used

Remember!
Ʃ(𝑥 − 𝑥̅ )2
𝑠2 =
𝑛

where: 𝑥 = individual score n = total number of scores


𝑥̅ = mean of all scores

To be able to apply the formula, the following steps ca be observed:

1. Arrange the values in column from lowest to highest.


2. Compute the mean of the distribution.
3. Determine the deviation (𝑑 = 𝑥 − 𝑥̅ ).
4. Square the deviations.
5. Get the sum of the squared deviations.
6. Divide the sum by the total number of cases. The quotient is the variance.

Example 18. Consider the data set below. Compute the variance of each data set.

Data Set
Data Set 1 13 16 14 10 15
Data Set 2 22 25 23 27 29

51
Solution:

Data Set 1:

Score Deviation Squared Deviation


𝒙 𝒙 − 𝒙 ̅ (𝒙 − 𝒙 ̅)𝟐
10 -3.6 12.96
13 -0.6 0.36
14 0.4 0.16
15 1.4 1.96
16 2.4 5.76
𝑥̅ = 13.6 Ʃ (𝑥 − 𝑥̅ )2 = 21.2

Ʃ(𝑥 − 𝑥̅ )2
𝑠2 =
𝑛

21.2
𝑠2 =
5

𝑠2 = 4.24

Data Set 2:
Ʃ𝑓(𝑥 − 𝑥̅ )2
𝑠2 =
𝑛

Where: f = class frequency


x = class mark
𝑥̅ = mean of the distribution
n = total number of cases or scores

Score Deviation Absolute Deviation


𝒙 𝒙 − 𝒙 ̅ (𝒙 − 𝒙 ̅)𝟐
22 -3.2 10.24
23 -2.2 4.84
25 -0.2 0.04
27 1.8 3.24
29 3.8 14.44
𝑥̅ = 25.2 Ʃ (𝑥 − 𝑥̅ )2 = 32.8

Ʃ(𝑥 − 𝑥̅ )2
𝑠2 =
𝑛

32.8
𝑠2 =
5

𝑠2 = 6.56

52
Variance for Grouped Data

When the data are presented in frequency distribution, the following formula must be
used

Remember!
Ʃ𝑓(𝑥 − 𝑥̅ )2
𝑠2 =
𝑛−1

where: 𝑥 = class mark n = total number of scores


𝑥̅ = mean of all scores f = frequency

To be able to apply the formula, the following steps ca be observed:

1. Compute the mean of the distribution.


2. Determine the deviation (𝑑 = 𝑥 − 𝑥̅ ).
3. Square the deviations.
4. Multiply the deviation by its corresponding frequency.
5. Add the results in step 4.
6. Divide the sum by n – 1. The quotient is the variance.

Example 19: Below is a frequency distribution of test scores of 50 students in Assessment of


Learning. Find the variance of the scores.

Class Interval Frequency


36 – 40 7
31 – 35 10
26 – 30 5
21 – 25 14
16 – 20 6
11 – 15 8
Ʃ𝑓 = 50

Solution: The mean of this distribution is 𝑥̅ = 25.40.

Class
f x ̅
𝒙 − 𝒙 ̅)𝟐
(𝒙 − 𝒙 ̅)𝟐
𝒇 (𝒙 − 𝒙
Interval
36 – 40 7 38 12.60 158.76 1,111.32
31 – 35 10 33 7.60 57.76 577.6
26 – 30 5 28 2.60 6.76 33.8
21 – 25 14 23 -2.40 5.76 80.64
16 – 20 6 18 -7.40 54.76 328.56
11 – 15 8 13 -12.40 153.76 1,230.08
𝑥̅ = 25.40 Ʃ𝑓 = 50 Ʃ𝑓(𝑥 − 𝑥̅ )2 = 3,362

53
Ʃf(x − x̅)2
s2 =
n−1

3,362
𝑠2 =
49

𝑠2 = 68.61
Example 20: Consider the frequency distribution below. Calculate the variance of the
distribution.

Class Interval Frequency (f)


33 – 37 6
28 – 32 9
23 – 27 12
18 – 22 8
13 – 17 10

Solution: The mean of this distribution is 𝑥̅ = 24.22.

Class
f x ̅
𝒙 − 𝒙 ̅)𝟐
(𝒙 − 𝒙 ̅)𝟐
𝒇 (𝒙 − 𝒙
Interval
33 – 37 6 35 10.78 116.2084 697.2504
28 – 32 9 30 5.78 33.4084 300.6756
23 – 27 12 25 0.78 0.6084 7.3008
18 – 22 8 20 -4.22 17.8084 142.4672
13 – 17 10 15 -9.22 85.0084 850.084
𝑥̅ = 24.22 Ʃ𝑓 = 45 Ʃ𝑓 (𝑥 − 𝑥̅ )2 = 1,997.778

Ʃf(x − x̅)2
s2 =
n−1

1,997.778
𝑠2 =
44

𝑠2 = 45.40

Standard Deviation for Ungrouped Data

Recall that, in the computation of the variance, the deviation was squared. This implies
that the variance is expressed in squared units. Extracting the square root of the value of the
variance will give the value of the standard deviation. In symbol, 𝑠.

To take the standard deviation of ungrouped data, extract the square root of the
variance. In mathematical formula,

Remember!
Ʃ(𝑥 − 𝑥)2
𝑠=√
𝑛

where: 𝑥 = individual score n = total number of scores


54
𝑥̅ = mean of all scores
Example 21. Consider the data set below. Compute the standard deviation of each data set.

Data Set
Data Set 1 13 16 14 10 15
Data Set 2 22 25 23 27 29
Solution:

Data Set 1:

By example 18, the variance of Data Set 1 is 𝑠2 = 4.24. Hence, the standard
deviation is

Ʃ(𝑥 − 𝑥)2
𝑠=√
𝑛

𝑠 = √4.24

𝑠 = 2.06

Data Set 2:

By example 18, the variance of Data Set 2 is 𝑠2 = 6.56 Hence, the standard
deviation is

Ʃ(𝑥 − 𝑥)2
𝑠=√
𝑛

𝑠 = √6.56

𝑠 = 2.56

On the basis of the obtained standard deviation, we say that the scores in Data Set 1
deviate from the mean by 2.06 units, on the everage. For Data Set 2, the scores deviate
from the mean by an average of 2.56 units.

Standard Deviation for Grouped Data.

To take the standard deviation of grouped data, extract the square root of the variance.
In mathematical formula,

Remember!
Ʃ(𝑥 − 𝑥)2
𝑠=√
𝑛−1

where: 𝑥 = individual score n = total number of scores


𝑥̅ = mean of all scores
55
Example 22: Below is a frequency distribution of test scores of 50 students in Assessment of
Learning. Find the standard deviation of the scores.

Class Interval Frequency


36 – 40 7
31 – 35 10
26 – 30 5
21 – 25 14
16 – 20 6
11 – 15 8
Ʃ𝑓 = 50
Solution:

By example 19, the variance is 𝑠2 = 68.61. Hence, the standard deviation is

Ʃ𝑓(𝑥 − 𝑥̅ )2
𝑠=√
𝑛

𝑠 = √68.61

𝑠 = 8.28

The scorese deviate from the mean by an average of 8.28 units.

Example 23: Consider the frequency distribution below. Calculate the variance of the
distribution.

Class Interval Frequency (f)


33 – 37 6
28 – 32 9
23 – 27 12
18 – 22 8
13 – 17 10

Solution:

By example 20, the variance of is 𝑠 2 = 45.40. Hence, the standard deviation is

Ʃ𝑓(𝑥 − 𝑥̅ )2
𝑠=√
𝑛−1

𝑠 = √45.40

𝑠 = 6.74

The scorese deviate from the mean by an average of 6.74 units.

56
LESSON 3:
MEASURES OF RELATIVE POSITION

Measures of Relative Position also referred as quantiles are descriptive measures


that locate the relative position of a score in relation to the rest in a given distribution. There
are three measures of relative position, namely: the percentile, deciles, and quartiles.

Quartiles (Q)

Quartiles are the score points which divides the distribution into four equal parts. Each
set of observations has 3 quartiles and are denoted by Q1, Q2, and Q3.

Q1 Q2 Q3

a. 25% of the distribution has a value ≤ Q1 (lower quartile or the first quartile).
b. 50% of the distribution has a value ≤ Q2 (median or middle quartile).
c. 75% of the distribution has a value ≤ Q3 (upper quartile or the last quartile).
Deciles (D)

Quartiles are the score points which divides the distribution into ten equal parts. Each
set of observations has 9 deciles and are denoted by D1, D2, D3, …D9.

D1 D2 D3 D4 D5 D6 D7 D8 D9

a. 10% of the distribution has a value ≤ D1 (first decile).


b. 50% of the distribution has a value ≤ D5 (median or fifth decile).
c. 90% of the distribution has a value ≤ D9 (9th decile).
Percentiles (P)

Percentiles are the score points which divides the distribution into one - hundred equal
parts. Each set of observations has 99 percentiles and are denoted by P1, P2, P3, …P99.

P10 P20 P30 P40 P50 P60 P70 P80 P90


0
a. 10% of the distribution has a value ≤ P10 (10th percentile).
b. 50% of the distribution has a value ≤ P50 (median or %0th percentile).
c. 90% of the distribution has a value ≤ P90 (90th percentile).
Relationship Among Percentile, Decile, and Quartile

57
 P10 = D1
 P20 = D2
 P25 = Q1
 P50 = D5 = Q1 = median
 P75 = Q3
 P90 = D9
Quantiles for Ungrouped Data

To compute the quantiles of ungrouped data, we will be using the (n + 1) method,


where n represents the total number of scores in a data set.

If (𝑛 + 1) 𝑥 𝑝 = 𝑗𝑔 where j is the integral part and g is the fractional part of the product,
then

Remember!

𝑃𝑉 = 𝑋𝑗 + 𝑔 (𝑋𝑗+1 − 𝑋𝑗 )

Thus, PV is the number in the jth position (Xj) of the ordered data plus g multiplied by
the difference between the succeeding value (Xj+1) and (Xj).

Example 24: Find Q1, D5, P80, and P99 for the following data:

45 67 78 55 88 90 56 68 99 40

65 70 86 99 59 75 45 84 69 50

Solution: Arrange first the data from lowest to highest

40 45 45 50 55 56 59 65 67 68

69 70 75 78 84 86 88 90 99 99

Solving for Q1.

Since Q1 = P25, we get 25% (20+1) = 5.25, which mean that j = 5 and g = .25
which means that Q1 is the 5th score (55) plus 0.25 of the difference between
the 6th score (56) and the 5th score (55). Hence,

𝑃𝑉 = 𝑋𝑗 + 𝑔 (𝑋𝑗+1 − 𝑋𝑗 )
𝑄1 = 55 + .25 (56 − 55)
𝑄1 = 55 + .25
𝑄1 = 55.25
Hence, 25% of the scores in the distribution are below 55.25.

Solving for D5.

58
Since D5 = P50, we get 50% (20+1) = 10.5, which mean that j = 10 and g
= .50 which means that D5 is the 10th score (68) plus 0.50 of the difference
between the 11th score (69) and the 5th score (68). Hence,

𝑃𝑉 = 𝑋𝑗 + 𝑔 (𝑋𝑗+1 − 𝑋𝑗 )
𝐷5 = 68 + .50 (69 − 68)
𝐷5 = 68 + .50
𝐷5 = 68.50

Hence, 50% of the scores in the distribution are below 68.50.

Solving for P80.

For P80, we get 80% (20+1) = 16.8, which mean that j = 16 and g = .80 which
means that P80 is the 16th score (86) plus 0.80 of the difference between the
17th score (88) and the 16th score (86). Hence,

𝑃𝑉 = 𝑋𝑗 + 𝑔 (𝑋𝑗+1 − 𝑋𝑗 )
𝑃80 = 86 + .80 (88 − 86)
𝑃80 = 86 + 1.60
𝑃80 = 87.60

Hence, 80% of the scores in the distribution are below 87.60.

Solving for P99.

For P99, we get 99% (20+1) = 20.79. Thus, P 99 is the score which is .79 of
the way from the 20th score to the next score. Since we do not have a score
beyond the 20th score we take the 20th score as the value of P99. Therefore,
P99 = 99.

Quantiles for Grouped Data

To compute the quantiles of grouped data, similar computation of the median will be
followed. Since Px is the value below which x% of the total number of cases lies, we can revise
𝑛
the formula for median by changing with x%(n). Therefore,
2

Remember!
𝑛
− 𝐹𝑏
𝑃𝑥 = 𝐿𝐿 + (2 )𝑐
𝑓
where:
LL = true lower limit of the class interval containing P x
Fb = the sum of all frequencies below the intervals
containing Px (or the <cf directly below the intervals
containing Px)
f = frequency of the intervals containing Px
c = class size;
n = total number of cases 59
Example 25. Consider the frequency distribution below. Find Q1, D4, and P90.

Class Interval F <cf


35.5 – 40.5 7 50
30.5 – 35.5 10 43
25.5 – 30.5 5 33
20.5 – 25.5 14 28
15.5 – 20.5 6 14
10.5 – 15.5 8 8
Ʃ𝑓 = 50

Solution:

Solving for Q1.

Since Q1 = P25, we first get 25%(n) to determine the interval class containing Q 1.
Note that 25% (50) = 12.5. With reference to the “<cf” column, 12.5 is between
8 and 14, so, the interval 15.5 – 20.5 contains P25. Thus, with reference to this
interval, we have LL = 15.5; Fb = 8; f = 6; and c = 5.

𝑥%(𝑛) − 𝐹𝑏
𝑃𝑥 = 𝐿𝐿 + ( )𝑐
𝑓

12.5 − 8
𝑄1 = 15.5 + ( )5
6

𝑄1 = 15.5 + 3.75

𝑄1 = 19.25

Hence, 25% of the scores in the distribution are below 19.25.

Solving for D4.

Since D4 = P40, we first get 40%(n) to determine the interval class containing D 4.
Note that 40% (50) = 20. With reference to the “<cf” column, 20 is between 14
and 28, so, the interval 20.5 – 25.5 contains D4. Thus, with reference to this
interval, we have LL = 20.5; Fb = 14; f = 14; and c = 5.

𝑥%(𝑛) − 𝐹𝑏
𝑃𝑥 = 𝐿𝐿 + ( )𝑐
𝑓

20 − 14
𝐷4 = 20.5 + ( )5
14

𝐷4 = 20.5 + 2.14

𝐷4 = 22.64

Hence, 40% of the scores in the distribution are below 22.64.

60
Solving for P90.

For P90, we first get 90%(n) to determine the interval class containing P90. Note
that 90% (50) = 45. With reference to the “<cf” column, 45 is between 43 and
50, so, the interval 35.5 –40.5 contains P90. Thus, with reference to this interval,
we have LL = 35.5; Fb = 43; f = 7; and c = 5.

𝑥%(𝑛) − 𝐹𝑏
𝑃𝑥 = 𝐿𝐿 + ( )𝑐
𝑓

45 − 43
𝑃90 = 35.5 + ( )5
7

𝑃90 = 35.5 + 1.43

𝑃90 = 36.93

Hence, 90% of the scores in the distribution are below 36.93.

Box Plots or “Box Whiskers Diagram”

A box plot, also known as Box Whiskers Diagram is a visual presentation of a


summary of a data set. This is obtained by plotting the values of the five descriptive statistics
of the data which are the smallest value, the lower quartile (Q 1), the median (Q2), the upper
quartile (Q3), and the largest value.

To construct a box plot, the following procedures must be considered:

1. Draw a box that spans from Q1 to Q3.


2. Draw line segment in the box that marks the median Q2.
3. Draw line segment (called whiskers) that extend from the box to the smallest and
largest values of the data.
Example 26: Construct a box plot for the following data:

40 45 45 50 55 56 59 65 67 68

69 70 75 78 84 86 88 90 99 99

Solution:

Based on the data, we have the following five summary values:

1. Smallest value: 40
2. Q1 = 55.25
3. Q2 = 68.50
4. Q3 = 85.5
5. Largest value: 99

61
The resulting box plot is:

S Q1 Q2 Q3 L

30 40 50 60 70 80 90 100

LESSON 4:
NORMAL DISTRIBUTION

Normal Distribution

The most common distribution we see in statistics is the normal distribution. This is
also known as the Gaussian distribution or the “bell curve”. Normal distribution is a family
curves determined by parameters µ (mean) and σ (standard deviation).

Mean = Median = Mode


Symmetrical sides

Asymptotic tail

The properties of the normal distribution


Source:are as follows:
thoughtco.com

1. It is bell – shaped and is symmetric with respect to the vertical line that passes
through the highest point of curve.
2. It is unimodal and the mean, median and mode are equal.
3. It is asymptotic with respect to the baseline, which means that the tails of the
distribution get closer and closer to the baseline without crossing the baseline.
4. The total area under the curve and above the baseline is always equal to 1.0.
Empirical Rule

Because the under the normal curve and above the baseline is 1.0, we consider the
normal curve as the graphic picture of the proportion of scores in a distribution. We state below
a common property of all normal curves with a given mean µ and standard deviation σ. This
property is called the empirical rule which highlights one interpretation of the standard
deviation as a concept of “distance”.

62
Source: statistics – made – easy.com

If a given set of data is assumed to be normally distributed with a given mean µ and
standard deviation σ, then

a. about 68.27% of all the cases are expected to fall between µ - σ and µ + σ.
b. about 95.45% of all the cases are expected to fall between µ - 2σ and µ + 2σ.
c. about 99.73% of all the cases are expected to fall between µ - 3σ and µ + 3σ.
Example 27: Suppose the first-year college class consisting of 120 students posted a mean
score of 70 with a standard deviation of 9 in their final exam in Math. Assuming
that the scores are continuously and normally distributed,

a. how many students are expected to score between 61 and 79?


b. within what two scores do we expect 95% of the students to score?
Solution:

a. Note that 61 = 70 – 9 = µ - σ. While 79 = 70 + 9 = µ + σ. According to the empirical


rule, 68% of the scores are expected to fall between µ - σ and µ + σ. Hence, about
68% (120) ≈82 pupils are expected to score between 61 and 79.

b. Again, from the empirical rule, we expect about 95% of the scores to fall between
the values µ - 2σ and µ + 2σ. Since, µ - 2σ = 70 – 2(9) = 52 and µ + 2σ = 70 + 2(9)
= 88, then, about 95% of the pupils are expected to score between 52 and 88.

Standard Scores (z-scores)

The standard score is the distance of the score from the mean in terms of the standard
deviation. It tells how many standard deviations the observed value lies above or below the
mean of its distribution. The standard score is useful in comparing observed values from
different distributions. To be able to find areas under the normal curve, observed values must
first be converted into standard scores, and these would help solve statistical problems.

To change an observed value into standard score, you use the following equation:

63
Remember!

𝑥 − 𝑥̅
𝑧 =
𝑠

where: 𝑥 = raw score / observed value s = standard score


𝑥̅ = mean

Note: A positive z-score will mean that the score/observed value is above the
mean.
A negative z-score will mean that the score/observed value is below
the mean.

Example 28. In a given distribution, the mean is 65 and the standard deviation is 6. Find the
corresponding standard score of:

a. 68 b. 59

Solution:

a. The corresponding standard score of 68 is

𝑥 − 𝑥̅ 68 − 65 3
𝑧68 = = = = 0.5
𝑠 6 6

which means that 68 is 0.5 standard deviation above the mean.

b. The corresponding z-score of 59 is

𝑥 − 𝑥̅ 59 − 65 −6
𝑧59 = = = = −1.0
𝑠 6 6

which means that 59 is 1 standard deviation below the mean.

Example 29: On the final examination in Math, the mean grade was 82 and the standard
deviation was 8. In English, the mean grade was 86 and the standard deviation
was 10. Joseph scored 88 in Math and 92 in English. In which subject was his
standing higher?

Solution: The first that has to be done us change the scores into standard scores.

For English
𝑥 −𝑥̅ 92 −86 6
𝑧𝐸 = = = = 0.6
𝑠 10 10

64
For Math
𝑥 −𝑥̅ 88 −86 6
𝑧𝑀 = = = = 0.75
𝑠 8 8

His standing in Math was higher than his standing in English. He was 0.6 standard
deviation above the mean in English and 0.75 standard deviation above the mean in
Math.

Standard Normal Distribution

A normal distribution can be converted into a standard normal distribution by


obtaining the z value. A z- value is the signed distance between a selected value, designated
x, and the mean, μ, divided by the standard deviation. It is also called as z scores, the z
statistics, the standard normal deviates, or the standard normal values. In terms of formula:

Remember!
𝑥 −𝜇
𝑧 =
𝜎

where: 𝑥 = raw score / observed value 𝜎 = standard deviation


𝜇 = mean of the distribution

The normal distribution property allows to compute a probability problem concerning x


into one corniness z. To determine the probability that x lies in a given interval, converting the
interval to a z scale and then compute the probability by using the standard normal distribution
table.

65
Example 30. Find the area under the standard normal curve between the mean and each
given value of z:

a. z = -1.33
b. z = 1.75

66
Solution:

a. To find the area between the mean z = 0 and z = -1.33, we read the z value of 1.3
on the first column, then the z value of 0.03 on the first row of Table 1. The
intersection of the identified row and column yields the number 0.4082.

0.4082

-1.33 0

Thus, the area from the mean up to the value of z = -1.33 is 0.4082 or 40.82%

b. For z = 1.75, we read the z value of 1.7 on the first column, then the z value of
0.05 on the first row. The intersection row and column yield the number 0.4599 or
45.99%

0.4599

Thus, the area from the mean up to the value of z = 1.75 is 0.4599 or 45.99%

Example 31: Find the area under the standard normal curve

a. to the left of z = 2.0


b. to the right of z = -1.0
c. to the right of z = 1.96
d. to the left of z = - 2.65
e. between z = 1.5 and z = 2.75
f. between z = -1.0 and z = 2.0

Solution:

a. The area to the left of z = 2.0 includes the area from z = 0 and z = 2.0 plus half of
the entire area under the normal curve. From the table, the area from the mean
up to z = 2.0 is 0.4772. Therefore, the entire area to the left of z = 2.0 is 0.5 +
0.4772 = 0.9772 or 97.72%

0.9772

0 2.0

67
b. The area to the right of z = -1.0 includes the area from the mean down to z = -1.0
plus half of the entire area under the normal curve. By symmetry, the area from
the mean down to z = -1.0 is equal to the area from the mean up to z = 1.0 which
is 0.3414. Thus, the entire area to the right of z = -1.0 is 0.5 + 0.3414 = 0.8414 or
84.14%

0.8414

-1.0 0

c. To find the area to the right of z = 1.96, we first note that the area from the mean
to the entire right is 0.5. If we subtract the area from the mean up to z = 1.96 from
0.5, we get the desired area to the right of z = 1.96. Using the normal table, the
area from the mean up to z = 1.96 is 0.4750. Therefore, the area to the right of z
= 1.96 is 0.5 – 0.4750 = 0.025 or 2.5%.

0.025

0 1.96

d. To find the area to the left of z = -2.56 is equal to the area to the right of z = 2.56
by symmetry. Using the normal table, the area from the mean up to z = 2.56 is
0.4960. Therefore, the area to the left of z = -2.56 is 0.5 – 0.4960 = 0.004 or 0.4%.

0.004

e. To find the area between z = 1.5 and z = 2.75, we get the area from the mean up
-2.56 0
to z = 2.75, then subtract the area from the mean up to z = 1.5. Using the normal
table, the area from the mean up to z = 2.75 is 0.4970 while the area from the
mean up to z = 1.5 is 0.4332. therefore, the desired area is given by 0.4970 –
0.4332 = 0.0638 or 6.38%.

0.0638

68
0 1.56 2.75

f. The area from z = - 1.0 to z = 2.0 can be obtained by adding the area from the
mean down to z = - 1.0 and the area from the mean up to z = 2.0. By symmetry,
the area from the mean down to z = - 1.0 is equal to the area from the mean
down to z = 1.0 which is 0.3414. Also, the area from the mean up to z = 2.0 is
0.4772. Therefore, the desired area is given by 0.3414 + 0.4772 = 0.8186 or
81.86%

0.8186

- 1.0 0 2.0

Application of Normal Distribution

Example 32. The average PAG-IBIG salary loan for RFS Pharmacy Inc. Employees is
₱23,000. If the debt is normally distributed with a standard deviation of ₱2,500,
find the probability that the employee owes less than ₱18,500.

Solution:

Step 1: Draw a figure and represent the area.

P(x < 18,500)

Step 2. Find the z value for ₱18,500.

x
23,000
,500  23,000  4,500
1818,500
z    1.80
 2,500 2,500

Step 3. Find the appropriate area. The area obtained in the Standardized Normal
Distribution Table is 0.4641, which corresponds to the area between z = 0
and z = -1.80.

P 1.80 z  0   0.4641

Step 4. Subtract 0.4641 from 0.5000.

P (x < 18,500) = P (z < -1.80)

= 0.5000 – P (-1.80 < z < 0)

69
= 0.5000 - 0.4641

= 0.0359

0.0359

18,500 23,000

Hence, the probability that the employee owes less than ₱18,500 in PAG-IBIG salary
loan is 0.0359 or 3.59%.

Example 33: The average age of bank managers is 40 years. Assume the variable is normally
distributed. If the standard deviation is 5 years, find the probability that the age
of a randomly selected bank manager will be in the range between 35 and 46
years old.

Solution: Assume that ages of bank managers are normally distributed; then cut off points
are as shown in the figure below.

Step 1. Draw a figure and represent the area.

35 40 46

Step 2. Find the two z values.

x 35  40  5 x 46  40 6
z    1.00 z    1.20
 5 5  5 5

Step 3. Find the appropriate area for z = -1.00 and z = 1.20 using the table.

P (-1.00 < z < 0) = 0.3413 P(0<z<1.20) = 0.3849

Step 4. Add P (-1.00 < z < 0) and P (0 < z < 1.20).

P (35 < x < 46) = P (-1.00 < z < 1.20)

70
= P (-1.00 < z < 0) + P (0 < z < 1.20)

= 0.3413 + 0.3849

= 0.7262

35 40 46 x - value
-1.00 1.20 z - value

Hence, the probability that a randomly selected bank manager is between 35 and 46
years old is 0.7262 or 72.62%.

Module5:
Logic

Introduction
In everyday life, reasoning proves different points. For instance, to prove your parents
that you performed well in school, you can show your grades. To prove your friends that you
are a true friend to them, you just need to be a loyal and honest friend. Similarly, mathematics
and computer science use mathematical logic or simply logic to prove results. To prove a
theorem in mathematics, we use mathematical logic. In computer science, logic is used to
prove results of computer algorithm or the correctness of a computer program.

Logic is commonly referred as the science of correct reasoning, especially regarding


making inferences. Mathematical reasoning and arguments are based on the rules of logic.

Learning Outcomes:
At the end of this chapter, you are expected to:

 Determine a given sentence as statement or not.


 Write compound statements using the language of logic.
 Create truth tables of given statements.
 Determine the truth value of a given statement.
 Write the converse, inverse, and contrapositive statements of conditional
statements.
 Determine whether statements are equivalent, tautologies or contradiction
using truth tables.
 Appreciate the nature and concept of71logic as a tool to prove results.
LESSON 1:
LOGIC STATEMENTS AND QUANTIFIERS

Logic Statements

A statement is a sentence either true or false

Remember!
If we can determine the truth or falsity of a statement, then it has a
truth value. If the statement is true, then the truth value of the statement is
true. But when its false, then the truth value is false also.

Example 1: Identify Statements

Determine whether each sentence is a statement.

1. Annabelle is beautiful.
2. Where do you live?
3. 1+1=2
4. x + 5 = 7.

Solution:

1. For most people that knows Annabelle they might agree on the statement but not
all will have the same thought specially people who don’t like Annabelle. Therefore,
statement one is not a logical statement.
2. The sentence “Where do you live?” is a question; it is not a declarative sentence.
Hence, it is not a statement.
3. Everyone knows that if 1 is added to 1 the answer will always be 2. Then, statement
2 is a logical statement.
4. x + 5 = 7 is a statement. It is known as an open statement, a sentence that has a
variable. The truth value of this statement will depends on the value of x. It is true
if x = 2, and it is false for any values of x. For any given x, it is true or false but not
both.

Simple and Compound Statements

A simple statement is a statement that conveys a single idea. A compound


statement is a statement that conveys two or more ideas. Connecting simple statements with
words and, or, if ...then, and if and only if creates a compound statement.

72
Example 2: Negation of a Statement

Give the negation of each statement.

1. p: A square is a rectangle.
2. q: Today is Monday.
3. r: Every student has a ballpen.

Solution:

1. ~p: A square is not a rectangle.


2. ~q: Today is not Monday
3. ~r: Not every student has a ballpen

73
Compound Statements and Grouping Symbols

In writing compound statement in symbolic form, we use parentheses to indicate that


simple statments are grouped.

74
In writing compound statement in words, we use comma to indicate that simple
statements are grouped together. Statements on the same side of a comma are grouped
together.

Example 5: Let p, q, and r represent the following statements:

p: Cathy’s dancing style is similar to Maja.


q: Cathy has a messy hair.
r: Cathy is a rock dancer.

1. Write (p ˄ q) → r as an english sentence.


2. Write “If Cathy is not a rock dancer, then Cathy does not have messy hair
and Cathy’s dancing style is not similar to Maja.” In symbolic form.

Solution:
1. If Cathy’s dancing style is similat to Maja and Cathy has a messy hair, then
Cathy is a rock dancer.
2. ~ r → (~ q ˄ ~ p )

75
Conjunction

Disjunction

A disjunction is a compound statement that is made by combining two or more


statements with the word “or” or with symbol ˄.

Example 6: Determine whether each statement is true or false.

1. 12 ≥ 10.
2. 6 is an even number and 6 is a composite number.
3. 5 is a prime number and 5 is an odd number.
4. 2 is an integer or 2 is an odd number.
5. – 5 ≤ - 10

Solution:

1. 12 ≥ 10 means 12 > 10 or 12 = 10. Since 12 > 10 is true, then the statement is true.
2. The statement is true because 6 is an even number at the same time composite
number.
3. The statement is true because 5 is a prime and odd number.

76
4. The statement “2 is an integer” is true and the statement “2 is an odd number” is
false. Applying disjunction rule, the statement is true.
5. – 5 ≤ - 10 means - 5 < - 10 or -5 = -10. Since both simple statements are false, the
truth value of the statement is false.

LESSON 2:
TRUTH TABLES, EQUIVALENT STATEMENTS, AND
TAUTOLOGIES

Truth Table

A truth table is a table used to check the “truth value” of any compound statement for
all possible truth values of its simple statements. It is a way to check all possible outcomes.
Presented below are the truth tables for negattion, conjunction, and disjunction for
review purposes.

77
Truth Table with Three or More Simple Statements

Compound statements that involve three or more simple statements require a standard
truth table form with 2n number of rows. For instance, if the statement involves 3 simple
statements, then the number of rows of truth table is 23 = 8. If 4 simple statements are involved,
then 24 = 16 number of rows must be produced, and so on.

78
Lastly, use the values of (p ˅ q) and r to produce the truth values of
(p ˅ q) ˄ r applying the truth value of conjunction. The shaded column is the truth
table for (p ˅ q) ˄ r.

79
Example 9. Construct a truth table for (p ˄ q) ˄ (~r ˅ q).

Solution: Using the procedure developed above, we can produce the following table. Te
shaded column is the truth table for p ˅ [~ (p ˄ ~q)]. The number below signigfies
the sequence in which columns were constructed.

Equivalent Statements

Two statemens are said to be equivalent if they both have the same truth value for all
possible truth values of their simple statements. The symbol ≡ is used to indicates equivalents
statements.
Example 9. Show that ~ ( p ˅ ~ q) and ~p ˄ q are equivalent statements.

Solution: Construct the truth table of the given statements and compare the results.

Since the truth values of the given statements are the same, then

~(p ˅ ~ q) ≡ ~p ˄ q.

80
Tautologies and Contradiction

A tautology is a statement that is “always true”. The opposite of a tautology is a


contradiction, a statement that is “always false”.

If you notice in the last column, it is all “Fs”. Thus, the statement is a self-
contradiction.

LESSON 3:
THE CONDITIONAL AND THE BICONDITIONAL

Conditional Statements

A conditional statement is a statement written in a form “If...then”. The phrase after


the word “if” is called antecedent. The phrase after the word “then “is called the consequent.

81
Example 12 . Determine the antecedent and consequent in the following statements.

1. If x + 5 = 8, then x = 3.
2. If a polygon hasfour sides, then it is a quadrilateral.
3. If you don’t study your lesson, then you will not pass the exam.
4. If 2 is an integer, then 2 is a rational number.

Solution:

1. Antecedent: x + 5 = 8
Consequent: x = 3

2. Antecedent: A polygon has four sides


Consequent: It is a quadrilateral

3. Antecedent: You don’t study your lesson


Consequent: You will not the pass the exam

4. Antecedent: 2 is an integer
Consequent: 2 is a rational number

Example 13 . Determine the truth value of each statement.

1. If 2 is an integer, then 2 is a rational number.


2. If a=1, b=2, c=3, then a + b + c > 5
3. If 3 + 5 = 8, then 8 – 3 = 5.

Solution:

1. The consequent is true, then the statement is true.


2. The consequent is false, then the statement is false.

82
3. Because both the antecedent and consequent are true, then the statement is
true.

Example 14 . Determine the truth value of each statement.

1. y + 7 = 12 if and only if y = 5
2. x2 = 25 if and only if x = 5
3. x > 4 if and only if x > 3.

Solution:
1. The statement is true since the statement “y + 7 = 12” will be only true if “y = 5”.
Otherwise, the statement will be false.
2. If x = -5, the first equation is true and the second equation is false. Thus, the
statement is false.
3. The statement is false. If x = 4, the first statement will become false.

LESSON 4:
THE CONDITIONAL AND RELATED STATEMENTS

Every conditional statement has three related statements. They are called the
converse, inverse, and contrapositive.

The Converse

Converse statement is a conditional statement that is formed by switching the


antecedent and the consequent of the conditional.

Remember!
The Converse of Conditional Statement

From p → q, converse statement will have q → p

83
The Inverse

An inverse statement is formed by negating both the antecedent and the consequent
of the conditional.

Remember!
The Inverse of Conditional Statement

From p → q, inverse statement will have ~p → ~q

The Contrapositive

Contrapositive is a conditional statement formed by negating both antecedent and


the consequent, and switching them. Contrapositive statements are inverse and converse at
the same time on a single conditional statement.

84
Observations:

1. Converse and inverse statements are equivalents because they have the same
truth values.
2. Conditional and contrapositive statements are equivalents because they have the
same truth values.

Example 15: Determine whether related statements are equivalent.

85
Module 6:
Apportionment and Voting

Introduction
N owadays, decision making is very vital. There are many situations where decisions
have to be made. For instance, when a family decides which house to buy, every member of
the family is given the chance to say something regarding the choices. During national and
local elections, people decide who will govern the country, province, municipality, or barangay.
Selecting baseball players to play in a game, best performer to receive an award, and even
deciding where to go for a summer vacation, or deciding foods to eat on special occasions. All
of these needs sound decision making. There are many ways of making the final decisions,
some simple, some more complex.

In this chapter we will look at several voting methods, weighted voting systems,and
how votes are apportioned or divided among voters or states.

Learning Outcomes:
At the end of this chapter, you are expected to:

 Solve problems involving the various systems of:


a. Apportionment
b. Voting
c. Weighted Voting
 Appreciate the importance of apportionment, voting, and weighted voting
systems.

LESSON 1:
INTRODUCTION TO APPORTIONMENT

86
Example 1. Andromeda, a small country consisting of 6 states is governed by 25
representatives. The number of representatives for each state is proportional to
the population of the state. The following table shows the population of each state
as to the last census.

State Population
Alpha 22,000
Beta 45,000
Gamma 36,000
Delta 15,000
Epzilon 75,000
Zeta 41,000
Total 234,000

Standard
State Population Quotient
Quota
22,000
Alpha 22,000 ≈ 2.35 2
9,360
45,000
Beta 45,000 ≈ 4.81 4
9,360
36,000
Gamma 36,000 ≈ 3.85 3
9,360

87
15,000
Delta 15,000 ≈ 1.60 1
9,360
75,000
Epzilon 75,000 ≈ 8.01 8
9,360
41,000
Zeta 41,000 ≈ 4.38 4
9,360
Total 234,000 22

Standard Number of
State Population Quotient
Quota Senators
22,000
Alpha 22,000 ≈ 2.35 2 2
9,360
45,000
Beta 45,000 ≈ 4.81 4 5
9,360
36,000
Gamma 36,000 ≈ 3.85 3 4
9,360
15,000
Delta 15,000 ≈ 1.60 1 2
9,360
75,000
Epzilon 75,000 ≈ 8.01 8 8
9,360
41,000
Zeta 41,000 ≈ 4.38 4 4
9,360
Total 234,000 22 25

Overall, Alpha, Beta, Gamma, Delta, Epzilon, and Zeta gets 2, 5, 4, 2, 8, and 4
representatives, respectively.

88
Example 2. Using the problem in example 1, determine the number of representatives for
each state using Jefferson plan.

Solution: The standard divisor is 9360. Hence, choose a modified divisor less than 9360. Let
say, 8300.

Standard
State Population Quotient
Quota
22,000
Alpha 22,000 ≈ 2.65 2
8,300
45,000
Beta 45,000 ≈ 5.42 5
8,300
36,000
Gamma 36,000 ≈ 4.34 4
8,300
15,000
Delta 15,000 ≈ 1.81 1
8,300
75,000
Epzilon 75,000 ≈ 9.04 9
8,300
41,000
Zeta 41,000 ≈ 4.93 4
8,300
Total 234,000 25

With Jefferson method, Alpha gets two representatives, Beta gets five representatives,
Gamma gets four representatives, Delta gets one representative, Epsilon gets nine,
and Zeta gets four representatives.

Huntington – Hill Apportionment Method

This method has been used by US Congress since 1940 to apportioned the members
of the Congress among the states. Huntington – Hill method is implemented by calcultating
what is called a Huntington – Hill number. This number can be computed by

89
Example 3. The table below shows the number of lifeguards that are assigned to three sites
of a certain beach resort and the number of rescues made by lifeguards at those
sites. Use the Huntington – Hill method to determine to which site a new lifegurd
shoud be assigned.

Number of Number of
Beach Sites
Lifeguards Rescues
A 25 1225
B 32 1516
C 18 975

Site A

(𝑃𝐴 )2 (1225)2
= ≈ 2,308.65
𝑎(𝑎 + 1) 25(25 + 1)

Site B

(𝑃𝐵 )2 (1516)2
= ≈ 2,176.38
𝑎(𝑎 + 1) 32(32 + 1)

Site C

(𝑃𝐶 )2 (975)2
= ≈ 2,779.61
𝑎(𝑎 + 1) 18(18 + 1)

Hence, the new lifeguard should be assigned to Beach Site C according to


the Huntington – Hill Apportionment Principle.

90
LESSON 2
INTRODUCTION TO VOTING

Voting, in a mathematical perspective, is the process of aggregating the preferences


of individuals in a way that attempts to describe the preferences of a whole group (Brilliant,
2020). Hence, voting is method for a group to make a collective decision or express opinion.
In this topic, we will discuss four different voting systems.

91
Fruits Rankings
James 2 1 3 4 2
Aljon 1 2 2 1 3
Apple 3 3 4 2 1
Carla 4 4 1 3 2
Number of Voters 15 17 12 9 7

Solution: Using the Borda count method, each first-place vote receives 4 points, each second
– place vote receives 3 points, 2 points for each third – place vote and 1 point for
fourth – place vote. The summary for each candidate is shown below.

James:

17 first – place votes 17 * 4 = 68


22 second – place votes 22 * 3 = 66
12 third – place votes 12 * 2 = 24
9 fourth – place votes 9*1 = 9
Total 167
Aljon:

24 first – place votes 24 * 4 = 96


29 second – place votes 29 * 3 = 87
7 third – place votes 7 * 2 = 14
0 fourth – place votes 0*1 = 0
Total 197
Apple:

7 first – place votes 7*4 = 28


9 second – place votes 9*3 = 27
32 third – place votes 32 * 2 = 64
12 fourth – place votes 12 * 1 = 12
Total 131
Carla:

92
12 first – place votes 12 * 4 = 48
7 second – place votes 7*3 = 21
9 third – place votes 9*2 = 18
32 fourth – place votes 32 * 1 = 32
Total 119

Using Borda count method, Aljon is the clear winner since he received the highest
total points.

Plurality with Elimination

As you have seen in the example of Plurality method, it is possible to produce a winner
even if majority of the voters did not vote for the winner. This situation can be resolved by
eliminating the candidates with the lowest votes and then holding another election. If another
election is done, naturally the supporters who voted for the eliminated candidate will have to
vote for their next choice. If the result does not produce a majority winner, then the process of
elimination is repeated until such time the winner had win by majority votes.

Example 6. A certain college is to implement a new logo. Members were asked to make
designs and submit them for voting. Among the five designs submitted, the 400
students are asked to rank the designs under consideration. The results are shown
in the following table.

Logo Rankings
Design 1 2 1 2 5 4
Design 2 1 2 3 2 5
Design 3 4 3 4 1 3
Design 4 3 5 1 4 2
Design 5 5 4 5 3 1
Number of Voters 47 70 108 80 95

Logo Rankings
Design 1 1 1 2 4 4
Design 3 3 2 3 1 3
Design 4 2 4 1 3 2
Design 5 4 3 4 2 1
Number of Voters 47 70 108 80 95

From the table, Design 1 has 117 first – place votes, Design 3 has 80 first -place
votes, Design 4 has 108 first – place votes, and Design 5 has 95 first – place votes.

93
Because Design 3 has the fewest first – place votes, it is eliminated from
consideration. The new preference schedule is shown below.

Logo Rankings
Design 1 1 1 2 3 3
Design 4 2 3 1 2 2
Design 5 3 2 3 1 1
Number of Voters 47 70 108 80 95

It can be noted that Design 1 has 117 first -place votes, Design 4 has 108 first place
votes, and Design 5 has 175 first – place votes. From this, Design 4 must be
eliminated. The new preference schedule is shown below.

Logo Rankings
Design 1 1 1 1 2 2
Design 5 2 2 2 1 1
Number of Voters 47 70 108 80 95

From this table, Design 1 received 225 first – place votes and Design 5 has 175 first
– place votes. Since Design 1 received a majority votes, therefore, Design 1 should
be adapted.

Example 7. A certain company decided to have a team building activity. There are four
proposed beaches as the venue of the activity: Canhugas, Rawis, Minasangay,
and Calicoan. The preference schedule cast by 150 employees is shown below.

Beach Resort Rankings


Canhugas 2 1 3 4 3
Rawis 4 3 1 2 4
Minasangay 3 4 2 1 2
Calicoan 1 2 4 3 1
Number of Voters 32 40 25 15 38

versus Canhugas Rawis Minasangay Calicoan


Canhugas

94
Rawis
Minasangay
Calicoan

 Canhugas versus Rawis

Canhugas was favored over Rawis on 32 + 40 + 38 = 110 votes


Rawis was favored over Canhugas on 25 + 15 = 40 votes

 Canhugas versus Minasangay

Canhugas was favored over Minasangay on 32 + 40 = 72 votes


Minasangay was favored over Canhugas on 25 + 15 + 38 = 78 votes

 Canhugas versus Calicoan

Canhugas was favored over Calicoan on 40 + 25 = 65 votes


Calicoan was favored over Canhugas on 32 + 15 + 38 = 85 votes

 Rawis versus Minasangay

Rawis was favored over Minasangay on 40 + 25 = 65 votes


Minasangay was favored over Rawis on 32 + 15 + 38 = 85 votes

 Rawis versus Calicoan

Rawis was favored over Calicoan on 25 + 15= 40 votes


Calicoan was favored over Rawis on 32 + 40 + 38 = 110 votes

 Minasangay versus Calicoan

Minasangay was favored over Calicoan on 25 + 15 = 40 votes


Calicoan was favored over Minasangay on 32 + 40 + 38= 110 votes

versus Canhugas Rawis Minasangay Calicoan


Canhugas Canhugas Minasangay Calicoan
Rawis Minasangay Calicoan
Minasangay Calicoan
Calicoan

95
It can be noted that, Calicoan has three wins, Minasangay has two wins, and Canhugas
has one win. Hence, Calicoan is the winning beach.

LESSON 3:
WEIGHTED VOTING SYSTEMS

96
There are many possible number of coalitions. This can be computed by using the
formula:

Remember!

𝑁𝑢𝑚𝑏𝑒𝑟 𝑜𝑓 𝑃𝑜𝑠𝑠𝑖𝑏𝑙𝑒 𝐶𝑜𝑎𝑙𝑖𝑡𝑖𝑜𝑛𝑠 𝑜𝑓 𝑛 𝑣𝑜𝑡𝑒𝑟𝑠 = 2𝑛 − 1

Example 8. Company J is owned by four individuals namely: James, Jerick, Jonathan, and
Jeff. James has 250 shares, Jerick has 375 shares, Jonathan has 200 shares,
and Jeff has 175 shares. The quota to pass a measure is 501. The weighted voting
system for this company is {501: 250, 375, 200, 175}. Determine the winning
coalitions and the critical voters for each winning coalition.

97
Remember!
𝑛𝑢𝑚𝑏𝑒𝑟 𝑜𝑓 𝑡𝑖𝑚𝑒𝑠 𝑣𝑜𝑡𝑒𝑟 𝑣 𝑖𝑠 𝑎 𝑐𝑟𝑖𝑡𝑖𝑐𝑎𝑙 𝑣𝑜𝑡𝑒𝑟
Banzhaf Power Index BPI(𝐯) =
𝑛𝑢𝑚𝑏𝑒𝑟 𝑜𝑓 𝑡𝑖𝑚𝑒𝑠 𝑎𝑛𝑦 𝑣𝑜𝑡𝑒𝑟 𝑖𝑠 𝑎 𝑐𝑟𝑖𝑡𝑖𝑐𝑎𝑙 𝑣𝑜𝑡𝑒𝑟

Example 9. In reference to Example 8, determine the Banzhaf power index for A, B, C, and
D.

2
BPI(𝐀) = = 0.17
12

For BPI (B):

6
BPI(𝐁) = = 0.50
12

For BPI (C):

2
BPI(𝐂) = = 0.17
12

For BPI (D):

98
2
BPI(𝐃) = = 0.17
12

Hence, Voter B has a higher Banzhaf power index.

References

Acelejado, M., Belecera, R., & Blay, B. (1999). Mathematics for the Millennium. DIWA
Scholastic Press Inc. Makati Philippines, Philippines.

Asaad, A., & Hailaya, W. (2001). Statistics as Applied to Education and Other Related Fields.
REX Book Store. Manila, Philippines.

Aufman, R., Lockwood, J., Nation, R., & Clegg, D. (2013). Mathematical Excursions, 3rd
edition. Brooks/Cole, CENGAGE Learning.

Aufman, R., Lockwood, J., Nation, R., Clegg, D., & Epp, S. (2018). Mathematics in the Modern
World Philippine Edition. Rex Bookstore, Inc.

Aufmann R., Lockwood J, et al. (2018). Basic College Mathematics an Applied Approach, 14th
edition. Cengage Learning Asia: Philippines

Aufmann R., Lockwood J, et al. (2018). Mathematics in the Modern World,14th edition.
Cengage Learning Asia: Philippines

Bolaños, A. (1997). Probability and Statistical Concepts: An Introduction. REX Book Store.
Manila, Philippines.

Brilliant (2020). Mathematics of Voting. Retrieved from https://brilliant.org/wiki/mathematics-


of-voting/.

Calingasan, R., Martin, M., & Yambao, E. (2018). Mathematics in the Modern World. Quezon
City: C & E Publishing Inc.

Carpio J. and Peralta B. (2018). “Mathematics in the Modern World”. Books Atbp. Publishing
Corp.

Esller, B. et. al. (2012). Basic Statistics Textbook-workbook. Grand books Publishing Inc.
Manila, Philippines

Gabino, P. (2015). Statistics Concepts and Procedures.

99
Gabuyo, Y., & Dy, G. (2013). Assessment of Learning II Textbook and Workbook. REX Book
Store. Manila, Philippines

Pagala R. (2013). Basic Mathematics. Mindshapers Manila, Philippines.

PH Civil Service Reviewer (2014). How to Solve Venn Diagram Part 2. Retrieved from
http://civilservicereview.com/2014/12/solve-venn-diagram-problems-part-2/

Shoenfield, J. R. (2018). Mathematical logic. CRC Press.

Sirug W. Ph.D. (2015). Basic Probability and Statistics. Mindshapers: Manila Philippines

Sirug, W. (2018). Math in the Modern World. MINDSHAPERS Co., Inc. Manila, Philippines

https://www.studypug.com/home.html

https://www.wright.edu/degrees-and/catalog/courses/mth - 1450

100

You might also like